th Answer Key & Exp for GS Prelim Test-12 held on 24 Feb’ 2019

1. Which of the following pairs is/are correctly matched? Crops in Feature 1. Wheat : It requires 210 frost-free days. 2. : It requires high temperature and bright sunshine. 3. Coffee : It is cultivated in the highlands. 4. Tea : It is cultivated on the lower slopes of Nilgiri hills. Select the correct answer using the code given below: (a) 1 only (b) 2 and 4 only (c) 2, 3 and 4 only (d) 1, 2, 3 and 4

Answer: (c) Explanation: Pair 1 is incorrect. 210 frost-free days are required for Cotton crop and not Wheat. Wheat is primarily a crop of temperate zone. Its cultivation in India is done during winter. Hence, it is a rabi crop. Around 85 per cent of the total area under this crop is concentrated in north and central of the country. , Punjab, , and are five leading wheat producing states in India. Pair 2 is correct. Cotton requires high temperature, light rainfall or irrigation, 210 frost- free days and bright sunshine for its growth. It is a Kharif crop and requires 6 to 8 months to mature. It grows well in drier parts of the black cotton soil of the . Major cotton-producing states are – , , Madhya Pradesh, etc. Pair 3 is correct. Coffee is a tropical plantation crop. Coffee is cultivated in the highlands of Western in , and . Karnataka alone accounts for more than two third of total production of coffee in the country. Pair 4 is correct. Tea is a plantation crop. It is grown over undulating topography of hilly areas and well drained soils in humid and sub-humid tropics. Tea is cultivated on the lower slopes of Nilgiri and . Source: Vajiram and Ravi - Yellow Book, Indian Geography, Chapter 9: Agriculture, Page no – 188

2. As per Census 2011 of India, highest out-migration of people is witnessed from which among the following states of India? (1) (2) Gujarat (3) Uttar Pradesh Select the correct answer using the code given below: (a) 1 only (b) 2 only (c) 1 and 3 only (d) 2 and 3 only

Prelim IAS Test Series (2019) – GS Test 12 (24.02.2019) 1 Geography 2 and Current Affairs Sep, Oct, Nov, Dec 2018

Answer: (c) Explanation: Migration is an important factor affecting the course of socio-economic development in India. Accelerated movement of people originating mainly from the rural and backward areas in search of employment has been one of the most important features of the labour market scenario in India during the post-Independence period. According to the World Economic Forum's report on 'Migration and Cities', interstate migration in India doubled between 2001 and 2011 compared to the previous decade. Census 2011 pegs the total number of internal migrants in the country (accounting for inter- and intra-state movement) at a staggering 139 million. Uttar Pradesh and Bihar are the biggest source states, followed closely by Madhya Pradesh, Punjab, Rajasthan, , Jammu and Kashmir and . The major destination states are , Maharashtra, Tamil Nadu, Gujarat, and Kerala. Source: Vajiram and Ravi - Yellow Book, Indian Geography, Chapter 15: Urbanization in India, Page no: 346

3. Consider the following statements with reference to the 'Indian Peninsular Plateau': (1) It supports less population than that of the northern . (2) It consists of mostly relicts and residual mountains. Which of the statements given above is/are correct? (a) 1 only (b) 2 only (c) Both 1 and 2 (d) Neither 1 nor 2

Answer: (c) Explanation: The Indian Peninsular Plateau lies to the south of the Indo-Gangetic and is flanked by the sea/ocean on three sides. The Peninsula is formed essentially by a great complex of very ancient and , which constitutes a major part of it. Statement 1 is correct: The Peninsular plateau supports less population than that of northern plain because of its rugged terrain and . The northern plain of India is densely populated because it is a large plain of alluvial soil. The deposition of alluvium in a vast basin lying at the foothills of the makes the northern plain very fertile. Statement 2 is correct: The Peninsular plateau consists of mostly relicts and residual mountains like the Aravalli hills, the , the Javadi hills, the Veliconda hills, the Palkonda range, the hills, etc. The plateau has senile topographic features like small rounded hills due to prolonged erosion. Source: Vajiram and Ravi Yellow Book, Indian Geography, Chapter 2: Physiography of India; Page no. 26

Prelim IAS Test Series (2019) – GS Test 12 (24.02.2019) 2 Geography 2 and Current Affairs Sep, Oct, Nov, Dec 2018

4. Consider the following map: Which of the area(s) marked in the map above (with X, Z and O letters) represent arid and semi-arid climatic zones of India? (a) X only (b) X and O only (c) Z and O only (d) X and Z only

Answer: (d) Explanation: The areas marked with ‘X’ and ‘Z’ letters represent the hot ‘arid and semi arid’ climatic zones respectively. The area marked as ‘O’ represents tropical wet regions of India. The term “arid” normally means a of the earth’s surface where rainfall is nil or inadequate. Hence, the vegetation is non- existent or sparse, agriculture difficult or impossible and human living conditions precarious. Generally, arid regions are those that receive less than 25 cm annual rainfall. It includes all hot and cold . These regions are devoid of any significant vegetation due to moisture stress. Regions receiving annual rainfall between 25 and 75 cm can be termed as semi arid regions.  The hot Indian arid zone lies in the north-west part of the India and comprise largely of the region of Rajasthan.  Semi-arid zones occur chiefly in the states of Rajasthan, Punjab, Haryana, Uttar Pradesh, Gujarat, Madhya Pradesh, Maharashtra, Karnataka, Andhra Pradesh and Tamil Nadu as shown in the map given above. Source: NCERT – Class XI, India – Physical Environment, Chapter: Structure and Physiography, Chapter – 2, Page no. 17

5. Consider the following features about a particular ‘Indian Wildlife species’: (1) It is found in the Little in Gujarat and its surrounding areas. (2) It is protected under Schedule I of the Wildlife (Protection) Act 1972. (3) It is included in Appendix I of the Convention on International Trade in Endangered Species. The description given above is most suitable for which one of the following: (a) Wild Ass (b) Sangai (c) (d)

Answer: (a) Explanation:

Prelim IAS Test Series (2019) – GS Test 12 (24.02.2019) 3 Geography 2 and Current Affairs Sep, Oct, Nov, Dec 2018

Indian Wild Ass is one of the sturdiest animals in the world withstanding scorching mid- day temperature up to 48° C or more without any shelter. It has the strength and speed of a horse and can run at the speed of 60-70 km/hr. The wild ass, locally known as ghudkhar and found only in the in Gujarat in India, has been classified as an endangered animal. It is protected under Schedule I of the Wildlife (Protection) Act 1972. It is included in Appendix I of the Convention on International Trade in Endangered Species (CITES), making international trade in this species illegal. Source: Vajiram and Ravi - Yellow Book, Indian Geography Chapter 6: Natural Vegetation and Wildlife, Page no. 106

6. Select the correct order of the meeting points (from East to West) of the following Ganga tributaries where they join the main stream of river Ganga? (1) Gomati (2) Gandak (3) (4) Kosi (5) Ghaghara Select the correct answer using the code given below: (a) 3-1-5-4-2 (b) 3-4-5-1-2 (c) 4-2-5-1-3 (d) 4-2-1-5-3

Answer: (c) Explanation: The correct order of the meeting points (from East to West) of Ganga tributaries where they join main stream of river Ganga is: Kosi - Gandak - Ghaghara - Gomati - Yamuna. The Ganga is the most important river of India both from the point of view of its basin and cultural significance. It rises in the Gangotri glacier near Gaumukh (3,900 m) in the Uttarkashi district of Uttaranchal. Here, it is known as the Bhagirathi. It cuts through the Central and the Lesser Himalayas in narrow gorges. The Son is its major right bank tributary. The important left bank tributaries are the Ramganga, the Gomati, the Ghaghara, the Gandak, the Kosi and the Mahanada. The river finally discharges itself into the near the Sagar Island. Note: The Yamuna, is the western most and the longest tributary of the Ganga. Source: NCERT class XI, India Physical Environment, Chapter – Drainage system, Chapter - 3, Page – 22

7. With reference to the soils found in India, consider the following pairs: Soil Region 1 Alluvial : Indo-

Prelim IAS Test Series (2019) – GS Test 12 (24.02.2019) 4 Geography 2 and Current Affairs Sep, Oct, Nov, Dec 2018

Soils Gangetic Plains 2 Laterite : Great Indian Soils Desert 3 Black : Deccan Trap Soils Which of the pairs given above is/are correctly matched? (a) 2 only (b) 1 and 2 only (c) 1 and 3 only (d) 1, 2 and 3

Answer: (c) Explanation: Pair 1 is correct. Alluvial Soils are the most common soils in India. These are widespread in northern plains and river valleys. These are rich in humus, lime and organic matter. These are highly fertile. Pair 2 is incorrect. The name of laterite soils is derived from Latin word ‘Later’ which means a Brick. These soils are most common in the areas of high temperature and high rainfall and are formed due to leaching. In India, laterite soils are found on the slopes of the and not in Indian desert. Pair 3 is correct. Black Soils are most common in Deccan Trap. These are mature soils with high water retaining capacity. These soils are ideal for cultivation of cotton. Source: NCERT Class XI, India: Physical Environment, Chapter 6, Page no 68-71.

8. Consider the following statements: (1) In India, Andhra Pradesh has the longest mainland coastline. (2) The coastal plains are one of the most densely populated areas of the world. Which of the statements given above is/are correct? (a) 1 only (b) 2 only (c) Both 1 and 2 (d) Neither 1 nor 2

Answer: (b) Explanation: The total length of India’s coastline is 7516 km. Of this, mainland coastline length is 5422 km whereas Island territories coastline length is 2094 km. Statement 1 is incorrect: The 1214.7 km long coastline of Gujarat, which is

Prelim IAS Test Series (2019) – GS Test 12 (24.02.2019) 5 Geography 2 and Current Affairs Sep, Oct, Nov, Dec 2018 approximately 23% of the country’s total mainland coastline, makes it the state with the longest mainland coastline. Andhra Pradesh has the second longest mainland coastline with a total length of 974 km. Statement 2 is correct: Coastal area covers more than 10% of the earth surface. Because of the economic benefits that accrue from access to ocean navigation, coastal fisheries, tourism, recreation and industrialization, human settlements are often more concentrated in the coastal zone than elsewhere. About 40% of the world’s population lives within 100 km of the coast. About 35% of Indians live within 100 km of the country’s coastline measuring 7517 km. Source: NCERT – Class XI, India – Physical Environment, Chapter – Structure and Physiography, Chapter no. 2, Page no. 18

9. A man residing in southern India visited Sarnath for his spiritual trip. He travelled mostly on a particular National Highway. Some details about his journey are mentioned below: (1) He started from southernmost point of peninsular India. (2) He got a chance to see Silicon Valley of India on the way. (3) He encountered 'Tiger Capital of India' during his journey. If these things occurred in a sequence as mentioned above t hen the man must have used which one of the following National Highways? (a) National Highway 5 (b) National Highway 6 (c) National Highway 7 (d) National Highway 8

Answer: (c) Explanation: National Highway 7 is renamed as National Highway 44, The 2,369 km long road is the longest National Highway of India and one of the major road network between south and . It connects to Kanyakumari. Sarnath is 10km from Varanasi. The National Highway 7 or 44 connects major Indian cities like Varanasi, , , , Madurai, Kanyakumari. Kanyakumari is the southernmost point of peninsular India. It is the meeting point of three seas/oceans-the Bay of Bengal, the and the . The Silicon Valley of India is a nickname for the Indian city of Bangalore. The name signifies Bangalore’s status as a hub for information technology (IT) companies in India. Nagpur is called as the Tiger Capital of India. Maharashtra has done exceptionally well in the growth of tigers - which has prompted government to promote Nagpur as the “Tiger Getaway” of India. Value Addition:  National Highway 8 - From Delhi to Dankuni  National Highway 6 - From Hajira to Kolkata  National Highway 5 - From to Source: Vajiram and Ravi - Yellow Book, Indian Geography Chapter 13: Transport and Communication Network, Page no. 279

10. Consider the following map:

Prelim IAS Test Series (2019) – GS Test 12 (24.02.2019) 6 Geography 2 and Current Affairs Sep, Oct, Nov, Dec 2018

Which of the following crops are cultivated under the areas as marked (with X and O letters respectively) in the India map given above? (a) Coffee, Bajra (b) Tea, Wheat (c) Jute, Groundnut (d) Coffee, Jute

Answer: (d) Explanation: Coffee: Commercial plantations of coffee started in India during the 18th century. Coffee grows in well drained soil, tropical temperatures, well distributed rains and shade (as sunshine breaks open the beans). Hence, the area under coffee plantations in India is concentrated in the southern states of Karnataka (53.83%), Kerala (18.89%) and Tamil Nadu (7.83%). Jute: Jute is soft, flexible and hard-wearing fibre. Jute fibres are long, lustrous, resilient and softer to touch. Jute crop requires humid with temperature fluctuating between 24 degree Celsius and 38 degree Celsius. Minimum rainfall required for jute cultivation is 100 cm. New grey alluvial soil of good depth receiving silt from annual floods is most suitable for jute growth. The cultivation of jute in India is mainly confined to the eastern region of the country. West Bengal alone accounts for over 50 percent raw jute production. The leading world's jute producing countries are India, Bangladesh, China and Thailand. India is the world's largest producer of raw jute and jute goods, contributing to over 50 percent and 40 percent respectively of global production. Source: NCERT XII - India, India People and Economy, Chapter 5: Land Resources and Agriculture, Page 53-54.

11. In the context of Indian states, consider the following statements: (1) Per capita income of is relatively more than that of Bihar. (2) Per capita consumption of electricity is relatively higher in Gujarat than in . Which of the statements given above correctly indicates the regional disparity in India?

Prelim IAS Test Series (2019) – GS Test 12 (24.02.2019) 7 Geography 2 and Current Affairs Sep, Oct, Nov, Dec 2018

(a) 1 only (b) 2 only (c) Both 1 and 2 (d) Neither 1 nor 2

Answer: (c) Explanation: Both the statements (1) and (2) correctly indicate the regional disparity in India. Statement 1 is correct. The most important indicator of regional disparity among different states of India is the difference in per capita state income figures. As per the report of the Ministry of Statistics and Programme Implementation (released on 20 Aug 2015), the state per capita income of Goa (Rs. 2,42,745) is highest in the country and of Bihar (Rs. 31,380) is lowest in the country. Statement 2 is correct. Per capita consumption of electricity is also another important indicator of regional disparities. States like Punjab, Gujarat, Haryana, Maharashtra etc., having higher degree of industrialisation and mechanisation of agriculture, have recorded a higher per capita consumption of electricity than the economically backward states like , Bihar, Odisha, Madhya Pradesh and Uttar Pradesh. Source: Vajiram and Ravi - Yellow Book, Indian Geography, Chapter 16: Regional development and Planning, Page no. 362

12. Arrange the following features in the order of their occurrence (from north to south) on the India Map: (1) (2) Bhor (3) (4) Palghat Gap Select the correct answer using the code given below: (a) 1-2-4-3 (b) 1-4-2-3 (c) 3-2-1-4 (d) 3-1-2-4

Answer: (d) Explanation: Zoji La is a high mountain pass in the Indian state of Jammu and Kashmir, located on National Highway 1 between Srinagar and in the western section of the Himalayan mountain range. It separates Kashmir Valley from Dras Valley. Rohtang Pass is a high mountain pass at an elevation of 3979 m above the sea level, located on the eastern of the Himalayas. The pass connects valley with Lahaul and Spiti which in turn provide access to Leh. is located near in Maharashtra. The highway passes through this gap. or Palghat Gap is a low mountain pass in the Western Ghats between Coimbatore in Tamil Nadu and Palakkad in Kerala. It has an average elevation of 140 metres. The pass is located between the Nilgiri Hills to the north and to the south.

Prelim IAS Test Series (2019) – GS Test 12 (24.02.2019) 8 Geography 2 and Current Affairs Sep, Oct, Nov, Dec 2018

Source: NCERT Class 11th, India: Physical Environment, Chapter 2: Structure and Physiography, Page 17.

13. With reference to ‘Wastelands in India’, consider the following statements: (1) Wastelands can be formed due to erosion by water but not due to anthropogenic factors. (2) Rajasthan has highest area under wasteland in India whereas Delhi has the least area. Which of the statements given above is/are correct? (a) 1 only (b) 2 only (c) Both 1 and 2 (d) Neither 1 nor 2

Answer: (b) Explanation: Wastelands are the lands which due to neglect or degradation, are not being utilized to their full potential. Such lands can result from inherent disabilities such as location, environment, chemical and physical properties, etc. They even suffer from imposed disabilities like management conditions. Statement 1 is incorrect. Wastelands can be formed due to erosion by water. This includes sheet, rill and gully erosion. Also, anthropogenic factors like mining, shifting cultivation, etc contribute to the formation of wastelands. Statement 2 is correct. Rajasthan tops the chart with 25% of its geographical area being wasteland, accounting for 18% of the wasteland nationally. Maharashtra, Madhya Pradesh, Andhra Pradesh, have significant percentage of wasteland. Delhi stands at the bottom of the pile with not an inch of wasteland. Source: NCERT Class 12th, India: People and Economy, Chapter 5: Land Resources and Agriculture, Page 41.

14. Which among the following factors can influence the growth of Indian population? (1) Total Fertility Rate (2) Migration (3) Literacy rate Select the correct answer using the code given below: (a) 1 and 2 only (b) 2 and 3 only

Prelim IAS Test Series (2019) – GS Test 12 (24.02.2019) 9 Geography 2 and Current Affairs Sep, Oct, Nov, Dec 2018

(c) 1 and 3 only (d) 1, 2 and 3

Answer: (d) Explanation: Point 1 is correct. India’s total fertility rate - a measure of the number of children born to a woman during her lifetime - was down from 5.9 in 1951 to 2.3 in 2011. It was 3.2 at the time when National Population Policy, 2000 was adopted. India is expected to reach replacement level fertility of 2.1 by 2020. So, India is actually close to stabilizing its population. Point 2 is correct. In countries like India, immigration plays a very small role in the population change. Although people from neighbouring countries like Bangladesh, Pakistan and Nepal migrate to India; at the same time Indians migrate to other countries like the US, , and the UK. During the 1971 war between India and Pakistan over Bangladesh, the immigration rate increased tremendously. Point 3 is correct. Literacy rate in India is defined as the total percentage of the population in the age group of seven years or above which can read and write with understanding. Compared to barely 18 percent of India’s population recorded as literate in the first Census after Independence, according to the 2011 Census, that proportion has gone up to 74 percent. Compared with illiterate young women, educated ones desire smaller families and generally manage to achieve that goal. Better access to family planning and contraception must go hand-in-hand with improved education. Source: Vajiram and Ravi - Yellow Book, Indian Geography, Chapter 14: Population, Page no: 306

15. Which of the following is the primary reason behind large scale ‘Coral Bleaching,’ recently witnessed off the coast of Andaman and Nicobar Islands? (a) Aggressive fishing techniques (b) Increase in siltation (c) Discharge of chemicals (d) Rise in sea-surface temperature

Answer: (d) Explanation: Corals support immense as around 25% of the world’s fish species are dependent on them. Corals can grow under narrow environmental conditions - temperature between 22 and 26°C, salinity between 32 and 38 parts per thousand with more than 90% water clarity. Coral bleaching, whitening of coral that results from the loss of a coral’s symbiotic (zooxanthellae) or the degradation of the algae’s photosynthetic pigment. Such bleaching is associated with the devastation of coral reefs. In 2016, there was a loss of more than 23% corals off the coast of Andaman and Nicobar Islands when the sea-surface temperature increased due to El-Nino effect. In a single year, the reef cover went down from 52.27% to 39.94% in India. Note: El-Niño effect is the irregular periodic warming of the eastern that affects the climate in the tropical and sub-tropical regions. Source: NCERT XI- India: Physical Environment, Chapter 4: Climate, Page no-38-53

Prelim IAS Test Series (2019) – GS Test 12 (24.02.2019) 10 Geography 2 and Current Affairs Sep, Oct, Nov, Dec 2018

16. The Narmada valley lies between which of the following mountain ranges? (a) Bhander and Maikal (b) Satpura and Aravalli (c) Satpura and Vindhyas (d) Vindhyas and Aravalli

Answer: (c) Explanation: The Satpura and Vindhaya ranges run parallel to each other. The is a mountain range that stretches through the central part of India. It extends across Maharashtra, Madhya Pradesh and . The geographically separates Northern India from Southern India as it runs across . It is a very old mountain range in Central India. It extends in east-west direction from Varanasi to Gujarat through Madhya Pradesh. Most of the major Peninsular Rivers except Narmada and Tapi flow from west to east. Narmada, the largest west flowing river of the peninsula, rises near range in Madhya Pradesh. It is the longest river of Indian Peninsula which drains into Arabian Sea. It is one of the rivers in India that flows in the rift valley, between Satpura and Vindhyas range. The Narmada flows in trough faults and fill the original cracks with their detritus materials. Hence, there is a lack of alluvial and deltaic deposits in this river. Source: Vajiram and Ravi - Yellow Book, Indian Geography, Chapter 3 – Drainage System and River Water, Chapter 3, Page no 44

17. India is rich in a few minerals but lacks some other important ones. In this context, consider the following statements: (1) India is the largest producer in the world. (2) India is the second largest producer of after Australia. Select the correct answer using the code given below: (a) 1 only (b) 2 only (c) Both 1 and 2 (d) Neither 1 nor 2

Answer: (d) Explanation: On the basis of chemical and physical properties, minerals are grouped as − Metallic minerals and Non-metallic minerals. Major examples of metallic minerals are ore, copper, , etc. Depending upon the origination, non-metallic minerals are either organic (such as fuels also known as mineral fuels, which are derived from the buried animal and plant, e.g. such as coal and ), or inorganic minerals, such as , , graphite, etc. Statement 1 is incorrect. India is 3rd largest producer of coal, 4th largest producer, and 3rd largest steel producer. It has 7th largest reserves of .

Prelim IAS Test Series (2019) – GS Test 12 (24.02.2019) 11 Geography 2 and Current Affairs Sep, Oct, Nov, Dec 2018

Statement 2 is incorrect. India is poor in deposits. (Panna diamond belt, Madhya Pradesh is one of the few areas where diamond is found). Value Addition: Among the mineral belts, North Eastern belts of Indian Peninsula are the richest mineral belt of India followed by the Central Belt. 90% of mineral deposits are found in Chota Nagpur belt. Source: Vajiram and Ravi Indian Geography Chapter 11: Minerals and Industries, Page no. 223-231

18. Match List-I with List-II and select the correct answer using the code given below: List-I List-II (Physical Feature) (Indian Region) A. Himalayas 1. Deccan B. Indo-Gangetic Plains 2. Rohilkhand C. Peninsular Plateau 3. Malabar D. West Coast Plains 4. Garhwal Codes A B C D (a) 4 1 2 3 (b) 1 4 3 2 (c) 1 2 3 4 (d) 4 2 1 3

Answer: (d) Explanation: Garhwal is the western region in the northern Indian state of Uttarakhand which is home to the Garhwali people. Lying in the Himalayas, it is bounded on the north by Tibet, on the east by Kumaon region, on the south by Uttar Pradesh state, and on the northwest by Himachal Pradesh state. The people of Garhwal are known as Garhwali and speak the Garhwali . Rohilkhand is a region which lies in north-western region of Uttar Pradesh state, named after the tribes. Rohilkhand lies on the upper alluvial plain. The Deccan region is a large plateau in western and southern India. It rises to 100 metres in the north, and to more than 1,000 metres in the south, forming a raised triangle within the 's coastline. It is between three mountain ranges and extends over eight Indian states. Malabar region refers to the geographic area of southwest India. It spans from the south- western coast of Maharashtra and goes along the coastal region of Goa, through the entire western coast of Karnataka and Kerala and reaches till Kanyakumari. It is flanked by the Arabian Sea on the west and the Western Ghats on the east. Source: NCERT Class 11th, India: Physical Environment, Chapter 2: Structure and Physiography, Page 8-16.

19. Which of the following factors influence ‘South-West ’ in India? (1) Intense heating of Tibetan plateau (2) Westerly Jet Stream (3) Tropical Easterly Jet Stream

Prelim IAS Test Series (2019) – GS Test 12 (24.02.2019) 12 Geography 2 and Current Affairs Sep, Oct, Nov, Dec 2018

Select the correct answer using the code given below: (a) 1 only (b) 2 and 3 only (c) 1 and 3 only (d) 1, 2 and 3

Answer: (d) Explanation: The south-west monsoon sets in over the Kerala coast by 1st June and moves swiftly to reach Mumbai and Kolkata between 10th and 13th June. By mid July, southwest monsoon engulfs the entire subcontinent. Various factors influence South-West Monsoon in India. Factor 1 is correct. The Tibetan plateau is intensely heated during summer and thereby establishes an atmospheric circulation that is conducive for the monsoon. As the vast Tibetan plateau, high up in the mountains, gets warmed up during the summer months, it heats the air above. Such warm air rises and hence creates an area of low pressure over Tibetan Plateau. This belt of low pressure sucks in moisture laden winds from the oceans, thus initiating the south-west monsoon. Factor 2 is correct. Sub-tropical Westerly Jet Stream is responsible for the onset of the monsoon. Until the end of May, this jet stream supplies the wind to the Gangetic plain which is heated up intensely as the sun is directly above the (during the summer in ). These winds supplied by this jet stream prevent the formation of a low pressure area in the Gangetic plain. When sub-tropical westerly jet stream moves towards north by the end of May, it no longer provides the winds to the Gangetic plain. This results in the formation of a Low Pressure area in the Gangetic plain, which starts attracting the South East Trade winds of the . When such winds enter the northern hemisphere, they bend by 90 degrees due to the change in direction of the Coriolis force in the northern hemisphere. This marks the onset of the South West Monsoon. Factor 3 is correct. The easterly jet stream steers the tropical depressions into India. These depressions play a significant role in the distribution of monsoon rainfall over the Indian subcontinent. The tracks of these depressions are the areas of highest rainfall in India. The frequency at which these depressions visit India, their direction and intensity, all go a long way in determining the rainfall pattern during the southwest monsoon period. Source: Vajiram and Ravi Geography Yellow book Indian Geography, Chapter 4: Climate, Page no. 64

20. Chambal basin is famous for which of the following features? (1) Rectangular drainage pattern (2) Badland topography (3) Jog waterfall (4) Bird's foot delta Select the correct answer using the code given below: (a) 1 only (b) 2 only (c) 2 and 3 only (d) 1, 2 and 3

Prelim IAS Test Series (2019) – GS Test 12 (24.02.2019) 13 Geography 2 and Current Affairs Sep, Oct, Nov, Dec 2018

Answer: (b) Explanation: Chambal basin is famous for badland topography. A region with a large number of deep gullies or ravines is called badland topography. Such ravines are widespread, in the Chambal basin. In case of and its many tributaries, river rejuvenation occurred not once or twice but multiple times. This along with the semi arid climate and the soil texture helped in creating a vast network of ravines in the Chambal river basin in Madhya Pradesh, Rajasthan and Uttar Pradesh. Chambal Badlands are extremely dissected, difficult to cross and agriculturally unfit. Source: NCERT XI – India: Physical Environment, Chapter 6: Soils, Page no 73

21. Consider the following statements regarding ‘’ of India: (1) They constitute flood spread over large stretches of land. (2) Because of their igneous origin, they lack . Which of the statements given above is/are correct? (a) 1 only (b) 2 only (c) Both 1 and 2 (d) Neither 1 nor 2

Answer: (a) Explanation: The Deccan Traps are one of the largest volcanic provinces in the world, located on the Deccan Plateau of west-central India. The term "trap" refers to the step-like hills forming the landscape of the region. Their formation began at the end of the period. Statement 1 is correct: Deccan Traps constitute multiple layers of solidified flood basalt that together are more than 2,000 m thick, formed through series of eruptions. Deccan Traps cover much larger than 50,000 km2 and these lava piles formed the plateau topography. Statement 2 is incorrect: The Deccan Traps are famous for the beds of fossils that have been found between layers of lava. Many animal and plant fossils have been found from the inter-trappean beds of the Deccan Traps. The lava flows, especially after the beginning and before the close of activity, passed through definite periods of no volcanic activity. During such periods; streams and lakes appeared on the cooled and congealed volcanic land forms and both animal and plant life appeared in these areas. This is indicated by the occurrence of inter-trappean layers found in the lower and upper flows and their fossil contents. Source: (1) Vajiram and Ravi - Yellow Book, Indian Geography Chapter 2: Physiography of India, Page no. 26 (2)http://volcano.oregonstate.edu/deccan-traps

22. In the last few years, India has made a quantum jump in ‘Solar Energy Capacity’. Consider the following statements in this regard: (1) Among Indian states, Rajasthan has the largest solar energy potential followed by Jammu & Kashmir. (2) At present, Tamil Nadu has the highest installed solar energy capacity among the Indian states.

Prelim IAS Test Series (2019) – GS Test 12 (24.02.2019) 14 Geography 2 and Current Affairs Sep, Oct, Nov, Dec 2018

Which of the statements given above is/are correct? (a) 1 only (b) 2 only (c) Both 1 and 2 (d) Neither 1 nor 2

Answer: (a) Explanation: India being a tropical country has 300 solar days and high potential for solar power. Statement 1 is correct. Among Indian states, Rajasthan (142.31 GW) has the largest solar energy potential followed by Jammu & Kashmir (111.05 GW). Statement 2 is incorrect. Karnataka is the top solar state in India. Karnataka represents 24 percent market share of the total installed solar energy capacity in India as of March 2018. Value Addition: Domestic manufacturing of solar cells and modules in India is being supported by the through Modified Special Incentive Package Scheme (M-SIPS) of the Ministry of Electronics & Information Technology. The scheme, inter alia, provides for:  20-25% subsidy for investments in capital expenditure for setting up of the manufacturing facility.  Reimbursement of counter-vailing Duty (CVD)/ Excise Duty for capital equipment for the units outside Special Economic Zone (SEZ). Source: Vajiram and Ravi Indian Geography Chapter 12: Energy Resources, Page no. 265

23. Why are the farmers in agricultural regions of Punjab and Haryana advised to add to the soil? (a) To kill the unwanted weeds from the crop lands. (b) To accelerate the process of nitrogen fixation in soil. (c) To solve the problem of salinity in the soil. (d) To increase the proportion of magnesium and sodium in the soil.

Answer: (c) Explanation: In the areas of intensive cultivation with excessive use of irrigation, especially green revolution areas like Punjab and Haryana, the fertile alluvial soils are becoming saline. Excessive irrigation with dry climatic conditions promotes capillary action, which results in the deposition of salt on the top layer of the soil. In such areas, farmers are advised to add gypsum to solve the problem of salinity in the soil. Source: NCERT XI – India: Physical Environment, Chapter 6: Soils, Page no 72

24. Arrange the following events in the order of their occurrence in India:

Prelim IAS Test Series (2019) – GS Test 12 (24.02.2019) 15 Geography 2 and Current Affairs Sep, Oct, Nov, Dec 2018

(1) Establishment of first Jute Mill (2) Introduction of Railways (3) Setting up of Tata Iron and Steel Plant in Jamshedpur (4) Start of first Coal Mine Select the correct answer using the code given below: (a) 1-2-4-3 (b) 4-1-2-3 (c) 4-2-1-3 (d) 1-4-2-3

Answer: (c) Explanation: Industrial development in India can be divided into two phases: ● 1947-1980 – control of the Government ● 1980-97 – Process of liberalisation took over Some landmark events were: ● Coal mining started at Raniganj in 1772 ● Railways were introduced in 1853 ● First Cotton textile mill at Mumbai in 1854 ● Jute Industry made its beginning with the first jute mill established in 1855 in the Hooghly Valley ● Tata Iron and Steel Plant was set up at Jamshedpur in 1907. Source: Vajiram and Ravi Yellow Book- Indian Geography, Chapter 11: Minerals and Industries, Page no. 237

25. Irrigation in India includes a network of canals, wells, tanks, etc. Which of the following sources has maximum contribution in irrigated regions of India? (a) Canals (b) Wells and Tubewells (c) Tanks (d) Multipurpose projects

Answer: (b) Explanation: Sources of irrigation in India: Wells and Tubewells – 60%. It is more widespread in plains, coasts and some regions of peninsular India. It is less costly and more flexible as water can be drawn whenever needed and ‘evaporation loss’ is minimised and there is no fear of “over irrigation”. Uttar Pradesh leads in well irrigation followed by Punjab, Haryana, Bihar, Gujarat and Andhra Pradesh. Canals – 30%. Canals are one of the main sources of irrigation in India. Canals are big water channels taken out from rivers to carry

Prelim IAS Test Series (2019) – GS Test 12 (24.02.2019) 16 Geography 2 and Current Affairs Sep, Oct, Nov, Dec 2018 water to places far away from the river. Punjab and Haryana have become the first granaries of country due to these canals. For example: Western Yamuna Canal, Sirhind Canal, etc. Tanks – 8%. It is prevalent in the uneven and relatively rocky plateau of Peninsular India. Tanks are commonly used in Deccan Plateau, Andhra Pradesh, Karnataka, Tamil Nadu and Maharashtra. Multipurpose projects – 4%. Example: Beas Project Source: Vajiram and Ravi Indian Geography Chapter 9: Agriculture, Page no. 177

26. Arrange the following Indian soil types in descending order of their coverage area (expressed as a percentage of Indian geographical area): (1) Black soil (2) Laterite soil (3) Arid soil (4) Alluvial soil Select the correct answer using the code given below: (a) 2-1-3-4 (b) 4-3-1-2 (c) 4-1-3-2 (d) 2-3-1-4

Answer: (c) Explanation: On the basis of genesis, colour, composition and location, the soils of India have been classified into: i. Alluvial soils ii. Black soils iii. Red and Yellow soils iv. Laterite soils v. Arid soils vi. Saline soils vii. Peaty soils viii. Forest soils Alluvial Soils: They are widespread in the northern plains and the river valleys. These soils cover about 40% of the total area of the country. They are depositional soils, transported and deposited by rivers and streams. Black Soil: They cover most of the Deccan Plateau which includes parts of Maharashtra, Madhya Pradesh, Gujarat, Andhra Pradesh and some parts of Tamil Nadu. These soils cover about 15% of the total area of the country. These soils are also known as the ‘Regur Soil’ or the ‘Black Cotton Soil’. Arid Soils: These soils are developed under arid and semi-arid conditions which occupy about 4.3% of the total area of the country. They are spread over Rajasthan, , Kachchh, Haryana and South Punjab. They range from red to brown in colour and are generally sandy in structure and saline in nature. Lower horizons of the soil are

Prelim IAS Test Series (2019) – GS Test 12 (24.02.2019) 17 Geography 2 and Current Affairs Sep, Oct, Nov, Dec 2018 occupied by ‘kankar’ layers which restricts the infiltration of water. Laterite Soil: They cover around 3.7% of the total area of the country. These are well developed on the summits of Western Ghats, , Rajmahal hills, Satpuras and Vindhyas. They also occur in lower levels and in valleys. The laterite soils develop in areas with high temperature and high rainfall characterised by alternate wet and dry season which leads to leaching away of siliceous matter of the rocks. Source: NCERT Class XI India Physical Environment– Chapter 6: Soils, Page no. 69- 70

27. Which of the following cities are located on latitudes, which are situated to the west of city Nagpur (State - Maharashtra)? (1) (2) (3) Hyderabad (4) Bengaluru Select the correct answer using the code given below: (a) 1 and 2 only (b) 1, 3 and 4 only (c) 2, 3 and 4 only (d) 1, 2, 3 and 4

Answer: (b) Explanation: Since Nagpur city is placed at the geographical centre of the country, the zero mile marker is located there. The distances to various destinations across the country are measured from this marker. The marker site was identified in the Great Trigonometrical Survey of India, undertaken by the Company and the British Administration. From the map given below, it is clear that Bhopal, Hyderabad and Bengaluru cities are located to the west of Nagpur city. Raipur is located to the east of Nagpur city.

28. With reference to ‘Lakes’ in India, consider the following statements: (1) Wular Lake is the largest freshwater lake in India. (2) Chilka Lake is the largest wintering ground for migratory birds in India. (3) Sambhar Lake is the largest inland saline lake of India. Which of the statements given above is/are correct? (a) 2 only (b) 1 and 2 only (c) 2 and 3 only (d) 1, 2 and 3

Answer: (d) Explanation: Wular Lake – Wular Lake is the largest freshwater lake in India and lies in the Kashmir Valley, 40 km northwest of Srinagar City in the Northwest of India. With a size of 189 sq. km, Wular Lake is also one of the largest freshwater lakes in .

Prelim IAS Test Series (2019) – GS Test 12 (24.02.2019) 18 Geography 2 and Current Affairs Sep, Oct, Nov, Dec 2018

Chilka Lake – Chilka Lake is located in eastern Odisha state of India. It is separated from the Bay of Bengal by a narrow spit. It is one of India’s largest saltwater lakes. Chilka Lake is the largest wintering ground for migratory birds, especially flamingos, on the Indian subcontinent. Sambhar Lake – Located around 75 kilometres from in Rajasthan, Sambhar lake is India’s largest inland saline lake and contributes significantly to the country’s salt production. The area turns home to an annual population of migratory birds from as far away as and also local migratory species that thrive and nest in its waters teeming with algae and small fish. Among these, the greater and lesser flamingos are perhaps the most popular. Source: Orient Black Swan School Atlas, Page no: 10

http://www.walkthroughindia.com/attraction/the-10-most-beautiful-lakes-in-india/

29. As per Census 2011, which of the following Indian States/UTs are categorized as most urbanized regions? (1) Goa (2) Delhi (3) Kerala Select the correct answer using the code given below: (a) 1 and 2 only (b) 2 only (c) 3 only (d) 1, 2 and 3

Answer: (d) Explanation: In Indian context, urbanisation is one of the greatest phenomena of the latter half of the 20th century. It began to accelerate after independence, due to the country's adoption of a mixed economy. The process of society’s transformation from a predominantly rural to a predominantly urban population is known as ‘Urbanisation’. Top 10 Urbanized States of India area as shown in the table given. Source: (1) Vajiram and Ravi - Yellow Book, Indian Geography, Chapter 15: Urbanization in India, Page no. 338 https://www.census2011.co.in/facts/topstateurban.html

Prelim IAS Test Series (2019) – GS Test 12 (24.02.2019) 19 Geography 2 and Current Affairs Sep, Oct, Nov, Dec 2018

30. Which among the following states have their population density more than the population density of India? (1) West Bengal (2) Kerala (3) (4) Haryana Select the correct answer using the code given below: (a) 1 and 4 only (b) 2 and 3 only (c) 1, 2 and 4 only (d) 1, 2 and 3 only

Answer: (c) Explanation: The population density of India has gone up to 382 persons per square kilometre in 2011 from 325 persons per square kilometre in 2001. States with very high density are located in Satluj-Ganga Plains, while mountainous Himalayan States have least population density. Peninsular States except Kerala (859) and Tamil Nadu (555) are marked with moderate density. Source: Vajiram and Ravi - Yellow Book, Indian Geography, Chapter 14: Population, Page no: 308

31. Gold deposits are found in India at only a few places. In this context, which of the following pairs are correctly matched? Field Location 1. Kolar field : Tamil Nadu 2. Ramagiri : Bihar field 3. Hutti field : Karnataka Select the correct answer using the code given below: (a) 1 and 2 only (b) 2 only (c) 3 only (d) None of the above

Answer: (c) Explanation: Gold is a precious metal but in India at present the reserves of gold ore are low. However, few regions where gold deposits can be found are as follows: Karnataka – Kolar, Hutti, Topuldedi and Wondalli. The Hutti gold mines are owned by the government of Karnataka. Kolar and Hutti fields together produce more than 98% of country’s total production of gold. Hence, pair 1 is incorrect but pair 3 is correct. Andhra Pradesh – The Ramagiri Gold field mine is situated in Anantpur district of Andhra Pradesh. Hence pair 2 is incorrect. Value Addition:

Prelim IAS Test Series (2019) – GS Test 12 (24.02.2019) 20 Geography 2 and Current Affairs Sep, Oct, Nov, Dec 2018

Jharkhand – River beds of Garranadi, Sonanadi, Subarnrekha Source: Vajiram and Ravi Indian Geography Chapter 11: Minerals and Industries, Page 228

32. The Amarkantak Hill is the source of two rivers flowing in two different directions. These rivers are (a) Narmada and Tapi (b) Narmada and Son (c) Tapi and Betwa (d) Tapi and Son

Answer: (b) Explanation: The Narmada, the largest west flowing river of the Peninsula, rises near Amarkantak range of mountains in Madhya Pradesh. The Son originates near Amarkantak in Madhya Pradesh, just east of the headwaters of the , and flows north-northwest through Madhya Pradesh state before turning sharply eastward. Source: Vajiram and Ravi - Yellow Book, Indian Geography Chapter 3: Drainage System and River Water, Page no. 39-44

33. India’s Western Ghats and Eastern Ghats meet at (a) (b) (c) Nallamalla Hills (d) Nilgiri Hills

Answer: (d) Explanation: The Nilgiri Hills, far south in Tamil Nadu is the meeting point of the Western Ghats and the Eastern Ghats. It contains several lofty peaks, the highest of which, , is 2634 meters above the sea level. Source: Vajiram and Ravi - Yellow Book, Indian Geography Chapter 2: Physiography of India, Page no. 23-25

34. Consider the following statements regarding the ‘Himalayan Mountain System’ of India: (1) The Greater Himalayas is the only range of the Himalayas which maintains its continuity from west to east. (2) Garo, are structurally a part of the Himalayas. Which of the statements given above is/are correct? (a) 1 only (b) 2 only (c) Both 1 and 2 (d) Neither 1 nor 2

Prelim IAS Test Series (2019) – GS Test 12 (24.02.2019) 21 Geography 2 and Current Affairs Sep, Oct, Nov, Dec 2018

Answer: (a) Explanation: The Himalayas consist of a series of parallel mountain ranges. Himalayas are not only the physical barrier; they are also a climatic, drainage and cultural divide. Statement 1 is correct: The Greater Himalayas (Himadri) is the northern-most and highest of all the Himalayan ranges. It is the only range of the Himalayas which maintains its continuity from west to east. The approximate length of the Great Himalayan range, also known as the central axial range, is 2,500 km from east to west, and their width varies between 160-400 km from north to south. Statement 2 is incorrect: Garo, Khasi and Jaintia in are structurally a part of peninsular plateau and not the Himalayan ranges. It is believed that due to the force exerted by the north-eastward movement of the at the time of the Himalayan origin, a huge fault was created between the Rajmahal hills and the Meghalaya plateau. Later, this depression got filled up by the deposition activity of the numerous rivers. Today, the Meghalaya and stand detached from the main Peninsular Block Source: Vajiram and Ravi Yellow Book, Indian Geography, Chapter 2: Physiography of India; Page no. 14-15-16-17

35. Match the names of major Indian ports with their locations (as shown in the map) Names: A. Kandla B. Ennore C. Paradip D. Cochin Code: A B C D (a) 1 2 4 3 (b) 1 4 2 3 (c) 4 3 2 1 (d) 3 2 4 1

Answer: (b) Explanation: Kandla Port is situated on the in of Gujarat. The Port of Kandla is the first special economic zone in India as well as in Asia. Ennore Port, officially renamed Kamarajar Port Limited, is located on the about 24 km north of Chennai Port, Chennai. It is the 12th major port of India. The Kamarajar Port is the only corporatized major port and is registered as a company. Paradip Port is an artificial, deep-water port of eastern coast of India which is located in the Odisha state. It is the major port situated at the confluence of great river and the Bay of Bengal.

Prelim IAS Test Series (2019) – GS Test 12 (24.02.2019) 22 Geography 2 and Current Affairs Sep, Oct, Nov, Dec 2018

Cochin port is one of the largest port in India and the major port on the Arabian Sea and Indian Ocean sea route. Source: Vajiram and Ravi - Yellow Book, Indian Geography, Chapter 13: Transport and Communication Network, Page no: 296

36. Consider the following statements with reference to ‘ Gas’: (1) Shale gas is a colourless and odourless gas. (2) Shale gas releases less CO2 than coal during combustion. (3) Andhra Pradesh has maximum number of shale gas blocks for exploration. Which of the statements given above is/are correct? (a) 1 and 2 only (b) 3 only (c) 2 and 3 only (d) 1, 2 and 3

Answer: (a) Explanation: For meeting the country’s growing energy needs, exploration and exploitation of shale gas is very essential. Statement 1 is correct. Shale gas is colourless, odourless gas. It is lighter than air. Statement 2 is correct. Shale gas releases 50% less CO2 than coal hence it is an environment friendly source for generating electricity. Statement 3 is incorrect. Maximum Shale gas blocks are in Gujarat (28) followed by Andhra Pradesh (10). Cambay, KG basin, Cauvery, Ganga and Assam-Arakan regions are also rich in Shale gas in India. Source: Vajiram and Ravi Yellow Book, Indian Geography, Chapter 12: Energy Resources, Page no. 258

37. Consider the following statements with reference to a particular ‘National Park’ of India: (1) It is a biodiversity hotspot in the Eastern Himalayas. (2) It is famous for a particular species of flying squirrel. (3) It is located in Arunachal Pradesh. The description given above is most appropriate for which one of the following Indian National Parks? (a) (b) (c) Keibul Lamjao National Park (d) Dudhwa National Park

Answer: (b) Explanation: Namdapha National Park, located in Arunachal Pradesh in north-east India is the largest protected area in the Eastern Himalaya biodiversity hotspot. It is also the third largest national park in India in terms of area. Namdapha flying squirrel is an arboreal, nocturnal flying squirrel endemic to north-eastern India where it is known from a single specimen collected in Namdapha National Park in 1981.

Prelim IAS Test Series (2019) – GS Test 12 (24.02.2019) 23 Geography 2 and Current Affairs Sep, Oct, Nov, Dec 2018

Source: Orient Black Swan School Atlas, Page no. 35

38. Consider the following statements about ‘ Island’: (1) Geomorphologically, the entire Majuli Island is a part of the alluvial flood plains of the . (2) Large scale deforestation is the primary challenge faced by the Majuli Island. Which of the statements given above is/are not correct? (a) 1 only (b) 2 only (c) Both 1 and 2 (d) Neither 1 nor 2

Answer: (b) Explanation: Assam's famed Majuli island, which recently won the title of being the world's largest river island, is declared India's first island district. The 880 square km Majuli island on the Brahmaputra River displaced Marajo island of Brazil to become the world's largest river island according to Guinness World Records. Statement 1 is correct: Geomorphologically, the entire Majuli Island is a part of the alluvial flood plains of the Brahmaputra River. The Island is formed of soil consisting mainly of silt deposits. Statement 2 is incorrect: Soil Erosion and Floods are the pressing challenges faced by the Majuli Island. Majuli had an area of 1,256 sq km in 1991 but due to soil erosion, it has now reduced to 875 sq km. The soil of Island is without cohesion and thus, susceptible to erosion. Also, Brahmaputra is one of the most difficult rivers to tackle as it is very dynamic due to frequent geomorphology changes. Flood control and anti-soil erosion measures adopted elsewhere may not be applicable in this river which is 3rd largest in the world and carries highest silt load amongst the rivers of its size. Source: NCERT XI, India – Physical Environment, Chapter 2: Structure and Physiography, Page no 13 – 14.

39. Consider the following Indian rivers: (1) Indus (2) (3) Brahmaputra (4) Indravati Which of the above rivers represent antecedent drainage system? (a) 1, 2 and 3 only (b) 2, 3 and 4 only (c) 1, 2 and 4 only (d) 1, 3 and 4 only

Answer: (d) Explanation: Antecedent streams are those which are originated prior to the upliftment of land

Prelim IAS Test Series (2019) – GS Test 12 (24.02.2019) 24 Geography 2 and Current Affairs Sep, Oct, Nov, Dec 2018 surface. In other words, antecedent streams antedate the upliftment of an upland or mountain across which they have maintained their present courses through continuous downcutting of their valleys. The nature and the rate of upliftment of land area is very important parameter for the development of antecedent drainage system. Examples of antecedent streams are found in almost all of the Fold Mountains of the world. Many of the major Himalayan rivers are the examples of antecedent streams e.g., the Indus, the Sutlej, the Ganga, the Ghaghra, the , the Gandak, the Kosi, the Brahmaputra etc. : It is a tributary of the river Godavari. Its starting point is found to be the Ghats of range, in the of the state of Odisha. It does not represent antecedent drainage system. Source: NCERT XI India: Physical Environment, Chapter 3: Drainage System, Page no 25, 26

40. Gorakhpur is the headquarters of which of the following Railway Zones of India? (a) South-Central Railway (b) North-Central Railway (c) North-Eastern Railway (d) East-Central Railway

Answer: (c) Explanation: For administrative convenience and efficient transportation, railways were grouped into railway zones. Gorakhpur is the headquarters of North-Eastern Railway Zone. Source: Vajiram and Ravi - Yellow Book, Indian Geography, Chapter 13: Transport and Communication Network, Page no: 287

41. The plain region at the foothills of the ‘Shiwalik Mountains’ where heavy boulders, and coarse sediments flown in by Himalayan rivers are deposited is called as (a) Tarai (b) Doon (c) Khadar (d) Bhabar

Answer: (d) Explanation: The northern plains of India are formed by the alluvial deposits brought by the Himalayan rivers – the Indus, the Ganga and the Brahmaputra. From the north to the south, the plains can be divided into three major zones: the Bhabar, the Tarai and the Alluvial plains. The flat plains along the sub-Himalayan region in North India are called Bhabar. It is a narrow belt ranging between 8-10 km parallel to the Shiwalik foothills. As a result of this, the streams and rivers coming from the mountains deposit heavy materials of rocks and boulders, and at times, disappear in this zone. South of bhabar is the tarai belt with an approximate width of 15-30 km where most of the streams and rivers re-emerge without having any properly demarcated channel, thereby creating marshy and swampy conditions.

Prelim IAS Test Series (2019) – GS Test 12 (24.02.2019) 25 Geography 2 and Current Affairs Sep, Oct, Nov, Dec 2018

Source: Vajiram and Ravi Yellow Book, Chapter 2: Physiography of India; Page no. 19

42. Which of the following places in India have known reserves of natural gas? (1) Assam (2) -Godavari Basin (3) Cauvery Basin Select the correct answer using the code given below: (a) 1 and 2 only (b) 2 and 3 only (c) 1 and 3 only (d) 1, 2 and 3

Answer: (d) Explanation: Gas is one of the cleanest fuels with less carbon dioxide per joule delivered than either by coal or oil and far fewer pollutants than other hydrocarbon fuels. The development of Natural Gas industry in the country started in 1960s with discovery of gas fields in Assam and Gujarat. After discovery of South Basin fields by ONGC in 1970s, Natural Gas assumed importance. Later discoveries were made in Gujarat, Krishna-Godavari basin, Cauvery basin, , Assam etc. Source: Vajiram and Ravi - Yellow Book, Indian Geography, Chapter 12: Energy Resource, Page no. 258

43. Which of the following species is/are found in forest located in West Bengal state of India? (1) Royal (2) Crocodile (3) Gangetic Dolphin Select the correct answer using the code given below: (a) 1 only (b) 1 and 2 only (c) 2 and 3 only (d) 1, 2 and 3

Answer: (d) Explanation: The Sundarbans is a cluster of low-lying islands in the Bay of Bengal, spread across India and Bangladesh, famous for its unique mangrove forests. This active delta region is among the largest in the world, measuring about 40,000 sq km. The Sundarban forest is about 10,000 sq km across India and Bangladesh, of which 40% lies in India, and is home to many rare and globally threatened wildlife species such as the estuarine crocodile, royal Bengal tiger (Panthera tigris), Water monitor lizard, Gangetic dolphin, and Olive Ridley turtle. Source: NCERT VII Chapter 8: Tropical and Subtropical Regions, Page no. 61 https://www.wwfindia.org/about_wwf/critical_regions/sundarbans3/about_sundarbans/

Prelim IAS Test Series (2019) – GS Test 12 (24.02.2019) 26 Geography 2 and Current Affairs Sep, Oct, Nov, Dec 2018

44. Which of the following pairs are correctly matched? River Tributary 1. Indus : Zaskar 2. Yamuna : Son 3. Godavari : Manjra 4. Krishna : Select the correct answer using the code given below: (a) 1 and 3 only (b) 1, 2 and 3 only (c) 2 and 4 only (d) 1, 3 and 4 only

Answer: (a) Explanation: A river basin is a part of land through which a main river and all its tributaries flows. It is the basic hydrological unit for planning and development of water resources of a country. Pair 1 is correctly matched: Some Himalayan tributaries of are Shyok, Gilgit, Zaskar, Hunza, Nubra, Shigar and Dras. The Chenab is the largest tributary of the Indus. Pair 2 is incorrectly matched: The tributaries of Yamuna River are Chambal, Sind, Betwa and Ken on its right bank, while Hindan, Rind, Sengar, Varuna, etc. join Yamuna on its left bank. Note: Son is a south bank tributary of the Ganga, originating in the Amarkantak plateau. Pair 3 is correctly matched: The principal tributaries of River Godavari are Penganga, Indravati, Pranhita, and Manjra. Pair 4 is incorrectly matched: The major tributaries of River Krishna are Koyna, Tungbhadra and Bhima. Note: Amravati is a tributary of River . Source: NCERT class XI, India Physical Environment, Chapter 3: Drainage System, Page no- 24-28

45. Consider the following statements with reference to ‘Tectonic Activity’ in India: (1) The Indian plate is moving at a speed of 1 km per year towards the north and north-eastern direction. (2) The Indian peninsula is tectonically stable as it has not witnessed any seismic activity in the past. Which of the statements given above is/are not correct? (a) 1 only (b) 2 only (c) Both 1 and 2 (d) Neither 1 nor 2

Answer: (c) Explanation: Statement 1 is incorrect. The Indian plate is moving at a speed of one centimetre per year towards the north and north-eastern direction and this movement of plates is being

Prelim IAS Test Series (2019) – GS Test 12 (24.02.2019) 27 Geography 2 and Current Affairs Sep, Oct, Nov, Dec 2018 constantly obstructed by the Eurasian plate from the north. As a result of this, both the plates are said to be locked with each other resulting in accumulation of energy at different points of time. Excessive accumulation of energy results in building up of stress, which ultimately leads to the breaking up of the lock and the sudden release of energy, causes an earthquake. Statement 2 is incorrect. High stresses generated by the continental collision, caused by the northward movement of India, have affected the entire Indian Peninsula. Due to intense pressure, the peninsula has up-warped in many places. And worse, ancient rifts that had criss-crossed the nation's sub-surface strata, are now suddenly being reactivated. Many geologists now believe that the 1993 earthquake was caused by the reactivation of just such a rift zone. The earthquake in Latur has shaken conventional theories. What it proves is that we can expect a 6.5 magnitude earthquake anywhere in the peninsula. It means that the so called Indian that extends from Kanyakumari to the Himalayan foothills is vulnerable to major seismic activity. Source: NCERT – XI India: Physical Environment, Chapter 7: Natural Hazards and Disasters, Page no. 80-81

46. A low density, yet uncontrolled, expansion of larger urban areas into surrounding agricultural areas is called as (a) Out Growth (b) Urban Agglomeration (c) Satellite Town (d) Urban Sprawl

Answer: (d) Explanation: An urban agglomeration is a continuous urban spread constituting a town and its adjoining outgrowths (OGs), or two or more physically contiguous towns together with or without outgrowths of such towns. An Out Growth (OG) is a viable unit such as a village or a hamlet or an enumeration block made up of such village or hamlet and clearly identifiable in terms of its boundaries and location. Satellite Towns are suburban centres providing cheaper and spacious alternative to main city. An Urban Sprawl is a low density uncontrolled expansion of larger urban areas into surrounding agricultural areas. It happens because of rapid growth of city’s population and widening range of economic activities. Source: Vajiram and Ravi - Yellow Book, Indian Geography, Chapter 15: Urbanization in India, Page no: 340

47. Which of the following pairs is/are correctly matched? Channel Feature 1. 10 degree : Separates the Andaman Islands and Nicobar Islands Channel 2. 9 degree Channel : Separates Minicoy Island from the rest of the 3. 8 degree Channel : Separates Great Nicobar island and Little Nicobar

Prelim IAS Test Series (2019) – GS Test 12 (24.02.2019) 28 Geography 2 and Current Affairs Sep, Oct, Nov, Dec 2018

Island Select the correct answer using the code given below: (a) 1 only (b) 1 and 2 only (c) 1 and 3 only (d) 2 and 3 only

Answer: (b) Explanation: Pair 1 is correct: The Ten Degree Channel separates the Andaman Islands and Nicobar Islands from each other in the Bay of Bengal. Pair 2 is correct: Minicoy Island is separated from Lakshadweep by 9 degree channel. Pair 3 is incorrect: Lakshadweep is separated from Maldives by 8 degree channel. Source: Vajiram and Ravi Yellow Book, Chapter 1: Indian: General Introduction; Page no. 6

48. Consider the following statements in the context of India’s geographical extent: (1) The mainland of India extends from Arunachal Pradesh in the east to Rajasthan in the west. (2) The north-south extent of India is more than its east-west extent. Which of the statements given above is/are correct? (a) 1 only (b) 2 only (c) Both 1 and 2 (d) Neither 1 nor 2

Answer: (b) Explanation: Statement 1 is incorrect. The mainland of India extends from Kashmir in the north to Kanniyakumari in the south and Arunachal Pradesh in the east to Gujarat in the west. India’s territorial limit further extends towards the sea upto 12 nautical miles (about 21.9 km) from the coast. Statement 2 is correct. In context of India’s extent, the actual distance measured from north to south extremity is 3,214 km, and that from east to west is only 2,933 km. Source: NCERT XI – India: Physical Environment, Chapter 1: India – Location, Page No. 2

49. Which of the following states is also known as the ‘Molassis Basin’? (a) (b) (c) (d) Meghalaya

Answer: (a) Explanation: Mizoram is a landlocked state in North East India. Mizoram is also known as the ‘Molassis basin’ which is made up of soft unconsolidated deposits. The state of Mizoram is popularly

Prelim IAS Test Series (2019) – GS Test 12 (24.02.2019) 29 Geography 2 and Current Affairs Sep, Oct, Nov, Dec 2018 known as the songbird of India and is one of the smallest states in the country. The name Mizoram signifies the land of the Lushai highlanders. It is located in the southern corner of nestled mainly between Myanmar and Bangladesh, but also shares a part of its boundaries with its neighbouring states of Assam, Manipur and Tripura. NCERT XI – India: Physical Environment, Chapter 2: Structure and Physiography, Page No. 15

50. With reference to fertilizers in India, consider the following statements: (1) Phosphatic and Potassic fertilisers have been decontrolled since 1992. (2) At a national level, Nitrogen (N): Phosphorous (P): Potassium (K) ratio of 4:2:1 is considered as an optimum fertilizer consumption ratio. Which of the statements given above is/are correct? (a) 1 only (b) 2 only (c) Both 1 and 2 (d) Neither 1 nor 2

Answer: (c) Explanation: Most of the fertilisers used in India have to be imported either in raw material or complete form. The burden of continuously rising prices of fertilisers falls to a great extent on the Government of India. Statement 1 is correct. In keeping with the policy of liberalisation and reforms, Phosphatic and Potassic fertilisers have been decontrolled since 1992. Statement 2 is correct. At a national level, the Nitrogen (N): Phosphorous (P): Potassium (K) ratio of 4:2:1 is considered as an optimum ratio. Value Addition: But the current consumption ratio of nitrogen, phosphorus and potassium (NPK) is 6.7:2.4:1. The situation is grimmer in major agricultural states like Punjab and Haryana where NPK use ratios are as high as 31.4:8.0:1 and 27.7: 6.1:1 respectively. The deterioration in fertilizer mix will not only have an impact on productivity of crops but also on long-term soil health. Source: Vajiram and Ravi Yellow Book, Indian Geography, Chapter 9: Agriculture, Page no. 175.

51. Canal irrigation is well developed in certain regions of India. In this context, which of the following pairs are correctly matched? Canal Area Served 1 Indira Gandhi Canal : Rajasthan 2 Buckingham Canal : West Bengal 3 Mutha Canal : Maharashtra Select the correct answer using the code given below: (a) 1 only (b) 1 and 3 only (c) 2 and 3 only (d) 1, 2 and 3

Prelim IAS Test Series (2019) – GS Test 12 (24.02.2019) 30 Geography 2 and Current Affairs Sep, Oct, Nov, Dec 2018

Answer: (b) Explanation: S. No. Canal Name Area served 1 Indira Gandhi Canal Punjab, Rajasthan 2 Buckingham Canal Andhra Pradesh, Tamil Nadu 3 Mutha Canal Maharashtra Pair 1 is correct. Indira Gandhi Canal, previously known as the Rajasthan Canal, is one of the largest canal systems in India. The canal project was launched on 31 March, 1958. The canal begins at the Harike Barrage - at the confluence of the Sutlej and Beas rivers in the Indian Punjab - and continues in a south-westerly direction for some 290 miles (470 km). Pair 2 is incorrect. Buckingham canal (258 km) along Pradesh is part of National Waterway (NW)-4. The Buckingham Canal is a manmade, saltwater, navigation canal that runs parallel to the Coromandel Coast in the north-south direction. Pair 3 is correct. Mutha Canal has been taken out from the river Mutha near Pune in Maharashtra. The canal starts from Khadakwasla, flows across the city and further travels towards Phursungi. Value Addition: ● Sampad Sagar Canal – Godavari ● Sirhind canal – Satluj ● Eden canal – Damodar ● Anicut Canal - Kaveri Source: Vajiram and Ravi Yellow Book, Indian Geography, Chapter 9: Agriculture, Page 179

52. Which of the following Indian rivers flows through a rift valley? (a) Bhima (b) Kaveri (c) Indravati (d) Damodar

Answer: (d) Explanation: The drains the eastern margins of the . It flows through a rift valley and finally joins the Hugli. The Damodar valley, along the Bihar–West Bengal border includes India’s most important coal and mica mining fields and has long been an area of active industrial development. The river is famous all over the world for the Damodar Valley Corporation (DVC). The length of the river is 592 km. The river traverses important cities like Asansol, Bokaro, Durgapur, etc. Note: Other rivers which flow through a rift valley are Narmada and Tapti. Source: Vajiram and Ravi - Yellow Book, Indian Geography, Chapter 3: Drainage System and River Water, Page no. 40

Prelim IAS Test Series (2019) – GS Test 12 (24.02.2019) 31 Geography 2 and Current Affairs Sep, Oct, Nov, Dec 2018

53. Which of the following represents the correct sequence of hills when you travel (from north to south) along ‘Eastern Ghats’ in India? (a) Javadi Hills – Anaimalai Hills – Cardamom Hills – Nallamala Hills (b) Nallamala Hills – Javadi Hills – Anaimalai Hills – Cardamom Hills (c) Javadi Hills – Nallamala Hills – Cardamom Hills – Anaimalai Hills (d) Nallamala Hills – Javadi Hills – Cardamom Hills - Anaimalai Hills

Answer: (b) Explanation: Nallamala Hills are located south of the . The hills run north to south, parallel to the Coromandel Coast on the Bay of Bengal. Javadi hills are located on the Eastern Ghats in North Arkat district of Tamil Nadu. Anaimalai Hills are extended in the north-south direction on the borders of Kerala and Tamil Nadu to the south of the Nilgiri Hills. Cardamom Hills are located to the south of the hills. Orient BlackSwan School Atlas – India Physical, Page No.10

54. Which of the following pairs is/are correctly matched? National Located in Parks of Indian State India 1. Kishtawar : Jammu and Kashmir 2. Dudhwa : Assam 3. Sariska : Rajasthan Select the correct answer using the code given below: (a) 1 only (b) 3 only (c) 1 and 2 only (d) 1 and 3 only

Answer: (d) Explanation: Pair 1 is correctly matched: Jammu and Kashmir - Salim Ali, Dachigam, Kishtawar, Hemis National Park. Kishtwar National Park: This national park

Prelim IAS Test Series (2019) – GS Test 12 (24.02.2019) 32 Geography 2 and Current Affairs Sep, Oct, Nov, Dec 2018 was basically made to protect the endangered species of snow and was given the status of a national park in 1981. It is located in the Kishtwar district of the state of Jammu and Kashmir. Pair 2 is incorrectly matched: Uttar Pradesh – Dudhwa National Park, Chandraprabha Wildlife Sanctuary. Dudhwa National park is located in the state of Uttar Pradesh. It was established as National Park in the year of 1977. This region comes under the belt. Pair 3 is correctly matched: Rajasthan - Desert, Keoladeo Ghana, Sariska, Ranathambor National Park. Sariska National Park: It is located in state of Rajasthan. The area of Sariska is rich in mineral resources, such as copper. banned mining in the area of Sariska National park in 1991. Sariska is the first tiger reserve in the world to have successfully relocated tigers. Few National Parks of North India: Orient BlackSwan School Atlas – India Physical, Page No. 35

55. With reference to ‘Drought in India’, Trikal is considered as most devastating due to its adverse effects. Which of the following can be included under such adverse effects? (1) Scarcity of food grains, fodder and water. (2) An epidemic due to spread of cholera, gastro-enteritis and heptatis. (3) Outward migration of productive population from villages. (4) Increase in soil erosion by wind, water and grazing animals. Select the correct answer using the code given below: (a) 1 and 3 only (b) 1, 2 and 3 only (c) 1, 3 and 4 only (d) 1, 2, 3 and 4

Answer: (b) Explanation: Droughts have cascading effects on various other aspects of environment and society. Point 1 is correct: Crop failure leading to scarcity of food grains (akal), fodder (trinkal), inadequate rainfall, resulting in shortage of water (jalkal), and often shortage in all the three (trikal) is most devastating. Point 2 is correct: Scarcity of water compels people to consume contaminated water resulting in spread of many waterborne diseases like gastro-enteritis, cholera, hepatitis, etc. Point 3 is correct: Large-scale death of cattle and other animals, migration of humans and livestock are the most common sight to be seen in the drought affected areas. Point 4 is incorrect: Due to inadequate rainfall, soil erosion is not possible by water. Source: NCERT XI – India: Physical Environment, Chapter 7: Natural Hazards and Disasters, Page 88-89-90

Prelim IAS Test Series (2019) – GS Test 12 (24.02.2019) 33 Geography 2 and Current Affairs Sep, Oct, Nov, Dec 2018

56. Which among the following can be found in the ‘Nilgiri Biosphere Reserve’ of India? (1) (2) Indian Bustard (3) Lion - Tailed Macaque (4) Red Panda Select the correct answer using the code given below: (a) 1 and 3 only (b) 2 and 3 only (c) 3 and 4 only (d) 2, 3 and 4 only

Answer: (a) Explanation: Point 1 is correct. The Nilgiri tahr inhabits the open montane grassland habitats at elevations from 1200 to 2600 m (generally above 2000 m) of the South Western Ghats. Their significant concentration is in the Nilgiri Hills, with smaller populations in the Anamalai Hills, , Palni Hills. Point 2 is incorrect. Historically, the great Indian bustard was distributed throughout , spanning 11 states, as well as parts of Pakistan. Its stronghold was once the in the north-west and the Deccan plateau of the peninsula. Today, its population is confined mostly to Rajasthan and Gujarat. Point 3 is correct. The lion-tailed macaque is an monkey endemic to the Western Ghats of . Point 4 is incorrect. In India, Red Panda is found in , western Arunachal Pradesh, Darjeeling district of West Bengal and parts of Meghalaya. It is also the state animal of Sikkim. Listed as Endangered in the IUCN red list of Threatened Species and under Schedule I of the Indian Wildlife (Protection) Act, 1972, the red panda has the highest legal protection at par with other threatened species. Source: Vajiram and Ravi - Yellow Book, Indian Geography, Chapter 6: Natural Vegetation and Wildlife.

57. Which of the following pairs is/are correctly matched? Indian River Origin 1. Kosi : Tribeni Ghat 2. Kaveri : Brahamagiri Hills 3. Chenab : Lahaul Spiti Select the correct answer using the code given below: (a) 1 only (b) 1 and 2 only (c) 2 and 3 only (d) 1, 2 and 3

Answer: (d) Explanation: Pair 1 is correct: originates from Tribeni Ghat in Nepal. The river Kosi consists of seven streams and is popularly called as Saptkoshi in Nepal. The river is braided and

Prelim IAS Test Series (2019) – GS Test 12 (24.02.2019) 34 Geography 2 and Current Affairs Sep, Oct, Nov, Dec 2018 shifts its course frequently. This has resulted in frequent devastating floods. Thus, the river is often termed as the “Sorrow of Bihar”. Pair 2 is correct: Kaveri River originates from Tala Kaveri in Brahamgiri hills located in Coorg plateau of Karnataka. The river descends from South Karnataka plateau to Tamil Nadu plains through famous Sivasamudram waterfall. The Kaveri has formed quadrilateral delta in Bay of Bengal. Pair 3 is correct: Chenab originates near Bara Lacha Pass in the Lahul Spiti part of Zaskar Range. The Chandra and Bhaga are the two main tributaries of river Chenab. Source: Vajiram and Ravi Yellow Book, Chapter 3: Drainage System and River Water; Page no. 39 and 44

58. Lake Loktak is famous for floating biomass islands i.e Phumdis. Phumdis, a rich source of vegetation is thinning out and even breaking away. The main reason behind this issue is (a) Siltation has decreased the water level in the lake. (b) Hydroelectric project constantly keeps high water level in the lake. (c) Large scale deforestation has caused an increase in surface run-off. (d) Unsustainable grazing by Hangul.

Answer: (b) Explanation: Lake Loktak (Manipur) stretches to about 500 sq. km during the rainy season and reduces to nearly half in the dry months of February and March. It is famous for the floating biomass islands or Phumdis. The Phumdis float during the rainy season. They sink during the dry months, sucking nutrients from the lake bed to replenish their roots and float again when the next monsoon cycle begins. This has been their life cycle for centuries but now they are at grave risk as the Loktak Hydroelectric Project constantly keeps the water level in the lake high. Unable to feed from the nutrients on the bed, the phumdis are thinning out and even breaking away. Worse, local residents themselves are breaking off pieces of the biomass to sell elsewhere as rich fish culture soil. Value Addition: In the heart of the Loktak Lake is the largest Phumdi, the 40 sq. km Keibul Lamjao National Park, home to the endangered brown-antlered deer, the sangai, whose habitat is under threat just as the islands are. Source: Orient Black Swan School Atlas, Page no. 35

59. Consider the following map:

Prelim IAS Test Series (2019) – GS Test 12 (24.02.2019) 35 Geography 2 and Current Affairs Sep, Oct, Nov, Dec 2018

Which of the following Indian rivers is represented by the stretch A-B as shown in the map? (a) Luni (b) Mahi (c) Betwa (d) Tapi

Answer: (a) Explanation: Luni originates from the of the in Rajasthan. Luni does have freshwater during the first hundred kilometres but as it reaches Balotra in Barmer, the water starts to turn saline from the rich salt content of the surface it flows on. Luni then flows into the Thar Desert and meets its end in Barine, located at the north-eastern part of the marsh called the Rann of Kutch in Gujarat, without flowing into any larger water body. Source: Orient Black Swan School Atlas, Page no. 10

60. The colour of which of the following soils is linked with the wide diffusion of iron in ancient crystalline and metamorphic rocks? (a) Black Soil (b) Laterite Soil (c) (d) Alluvial Soil

Answer: (c) Explanation: Red soil is an important soil resource, which bears substantial implication for sustainable development of agriculture and healthy growth of economy. Red soil contains a high percentage of iron content, which is responsible for its colour. It is formed by the weathering of ancient crystalline and metamorphic rocks, particularly acid granites and gneisses, quartzitic rocks, and felspathic rocks. Source: Vajiram and Ravi - Yellow Book, Indian Geography, Chapter 5: Land and Soil Resources, Page no. 90

Prelim IAS Test Series (2019) – GS Test 12 (24.02.2019) 36 Geography 2 and Current Affairs Sep, Oct, Nov, Dec 2018

61. Match List I with List II and select the correct answer by using the codes given below: List I List II (Tree (State/UT) Species) A. 1. Andaman & Nicobar Islands B. Deodar 2. Rajasthan C. Mahogany 3. Odisha D. Khair 4. Himachal Pradesh Codes: A B C D (a) 1 3 2 4 (b) 3 4 1 2 (c) 2 4 1 3 (d) 4 3 1 2

Answer: (b) Explanation: Forest Type Tropical Evergreen Forests Trees: Rosewood, Mahogony, Aini, Ebony, etc. Location: Western slope of the Western ghats, hills of the north-eastern region and the Andaman and Nicobar Islands. Tropical Moist Deciduous Forests Trees: Teak, sal, shisham, hurra, mahua, amla, semul, kusum, sandalwood, etc. Location: North-eastern states along the foothills of Himalayas, eastern slopes of the Western Ghats and Odisha. Tropical Thorn Forests Trees: Khair, neem, khejri, palas, Tussocky grass, etc. Location: Semi-arid areas of south west Punjab, Haryana, Rajasthan, Gujarat, Madhya Pradesh and Uttar Pradesh. Montane Forests Trees: Oak, chestnut, chir pine, Deodar, chinar, walnut, blue pine, spruce, silver firs, junipers, pines, birch and rhododendrons, etc. Location: Himalayan ranges which include states of Jammu and Kashmir, Himachal Pradesh, Uttarakhand, North-eastern States. Southern mountain forests include the Western Ghats, the Vindhyas and the Nilgiris. Source: NCERT India Physical Environment Class XI – Chapter 5, page no. 57-61

Prelim IAS Test Series (2019) – GS Test 12 (24.02.2019) 37 Geography 2 and Current Affairs Sep, Oct, Nov, Dec 2018

62. Arrange the following mountain ranges of ‘Kashmir Himalayas’ in order of their occurrence (from north to south) on the map of India. (1) Pir Panjal (2) (3) (4) Zaskar Select the correct answer using the code given below: (a) 4 – 3 – 1 – 2 (b) 1 – 4 – 3 – 2 (c) 1 – 3 – 4 – 2 (d) 2 – 3 – 4 – 1

Answer: (d) Explanation: Ranges from north to south are as follows: Karakoram – Ladakh – Zaskar – Pir Panjal. Kashmir or North-Western Himalayas: It comprises a series of ranges such as the Karakoram, Ladakh, Zaskar and Pir Panjal. The Kashmir Himalayas are also famous for Karewa formations, which are useful for the cultivation of Zafran, a local variety of saffron. Karakoram Range: K2, the second highest peak of the world, at 8,611m lies in the Karakoram Range. Hindu-Kush, an extension of the Karakoram Range runs in Afghanistan. Ladakh range: It forms northern wall of the Indus valley and the southern wall of the Nubra and Shyok valleys. It extends down into Tibet. Zaskar range: It forms northern wall of the Zaskar and Suru valleys and the southern wall of the Indus valley. Pir Panjal Range: It is a group of mountains in the Inner Himalayan region, running across the Indian states of Himachal Pradesh and Jammu and Kashmir. Source: Vajiram and Ravi Yellow Book, Chapter 1: Indian- General Introduction; Page no. 13

63. With reference to ‘Mangroves’, consider the following statements: (1) Most extensive area of mangroves is found in South East Asia. (2) India accounts for nearly 3.3% of the world’s mangrove vegetation. (3) Sundarbans accounts for half of the total area under Mangroves in India. Which of the statements given above is/are correct? (a) 3 only (b) 1 and 2 only (c) 1 and 3 only (d) 1, 2 and 3

Answer: (d) Explanation: Mangroves are plants that survive high salinity, tidal regimes, strong wind velocity and muddy anaerobic soil – a combination of conditions hostile for other plants. Not all coastal areas are suitable for mangrove plantation as mangroves require an appropriate mix of saline and freshwater and soft substrate like mudflats to be able to grow and perpetuate.

Prelim IAS Test Series (2019) – GS Test 12 (24.02.2019) 38 Geography 2 and Current Affairs Sep, Oct, Nov, Dec 2018

Statement 1 is correct: Most extensive area of mangroves is found in South East Asia (33%) followed by (15%). The total Mangrove cover in the world is 150,000 sq.km approximately. Statement 2 is correct: Mangroves are spread over an area of 4921 sq. km in India which is nearly 3.3 % of the world’s mangrove vegetation. Statement 3 is correct: Sundarban accounts for half of the total area under Mangrove in India. The Sundarban mangrove forest lies on the delta of the Ganges, Brahmaputra and Meghna rivers on the Bay of Bengal. Source: (1) Vajiram and Ravi - Yellow Book, Indian Geography, Chapter 6: Natural Vegetation and Wildlife, Page no. 99 (2) http://fsi.nic.in/isfr2017/isfr-mangrove-cover-2017.pdf

64. Which of the following Indian rivers is also known as Dakshin Ganga? (a) Kaveri (b) Godavari (c) Tungbhadra (d) Krishna

Answer: (b) Explanation: Popularly known as the Dakshin Ganga, the Godavari is the largest peninsular river system. The Godavari originates from Nasik district of Maharashtra and discharges its water into the Bay of Bengal. With total 1,465 km length, Godavari covers the areas of Maharashtra, Madhya Pradesh, Chhattisgarh, Odisha, etc. The Penganga, the Indravati, the Pranhita, and the Manjra are the major tributaries of Godavari.

Source: NCERT XI India: Physical Environment, Chapter 3: Drainage System, Page no. 27

65. Madhya Pradesh has the largest forest cover in the country in terms of absolute area. In terms of percentage of forest cover with respect to the total geographical area, which of the following Indian states/ U.Ts has the highest forest cover? (a) Arunachal Pradesh (b) Mizoram (c) Andaman and Nicobar Island (d) Lakshadweep

Answer: (d) Explanation:

Prelim IAS Test Series (2019) – GS Test 12 (24.02.2019) 39 Geography 2 and Current Affairs Sep, Oct, Nov, Dec 2018

Madhya Pradesh (77,414 sq. km) has the largest forest cover in the country in terms of area, followed by Arunachal Pradesh (66,964 sq. km) and Chhattisgarh (55,547 sq. km). In terms of percentage of forest cover with respect to the total geographical area, Lakshadweep with (90.33%) has the highest forest cover, followed by Mizoram (86.27%) and Andaman and Nicobar Island (81.73%). Source: (1) Vajiram and Ravi - Yellow Book, Indian Geography, Chapter 6: Natural Vegetation and Wildlife, Page no. 103 (2) http://pibphoto.nic.in/documents/rlink/2018/feb/p201821201.pdf

66. Which of the following species has witnessed a catastrophic population decline in the past due to use of livestock drug ‘Diclofenac’? (a) Indian Vulture (b) Indian Bustard (c) Indian Pitta (d) Indian Swiftlet

Answer: (a) Explanation: Widespread use of diclofenac in Indian cattle is linked to the deaths of millions of vultures that ate carcasses containing the drug, causing their populations to decline by more than 99% since the 1990s. Diclofenac is toxic to vultures even in small doses, causing kidney failure. Source: https://www.ncbi.nlm.nih.gov/pubmed/29159701

67. The maximum rainfall during the ‘South-West Monsoon’ season is received by which of the following regions of India? (a) Western Ghats (b) Deccan Plateau (c) Tamilnadu Coast (d) Southern Ranges of the

Answer: (d) Explanation: By early June, the inflow of the south-west monsoon into India brings about a change in the weather. The maximum rainfall of this season is received in the north-eastern part of the country. in the southern ranges of the Khasi Hills receives the highest average rainfall in the world. Early in the season, the windward side of the Western Ghats receives very heavy rainfall i.e more than 250cm. The Deccan Plateau and parts of Madhya Pradesh also receive some amount of rainfall in spite of lying in the area. Tamilnadu coast receives rainfall during the season of retreating monsoon. Source: NCERT Class IX – Contemporary India, Chapter 4: Climate, Page no. 33

68. Which among the following cash crops are grown in India? (1) Tobacco (2) Wheat (3) Cotton

Prelim IAS Test Series (2019) – GS Test 12 (24.02.2019) 40 Geography 2 and Current Affairs Sep, Oct, Nov, Dec 2018

(4) Select the correct answer using the code given below: (a) 1 and 2 only (b) 1 and 3 only (c) 1, 2 and 4 only (d) 1, 3 and 4 only

Answer: (b) Explanation: Agricultural crops which are grown with an objective of making profit are called as Cash Crops. Such crops have a good demand and a ready market for sale. In India, cash crops like , tobacco, tea, rubber, coffee, cotton etc are grown. Note: Wheat is a Rabi crop and Rice is Kharif crop. Source: NCERT Class VII, Chapter 8: Tropical and Subtropical regions, Page no: 61

69. Identify the biosphere reserves which are marked (as 1, 2 and 3) on the India Map given below.

Select the correct answer using the code given below: (a) Sesachalam Hills, Achanakmar-Amarkantak, Dibru Saikhowa (b) Nilgiri, Simlipal, Dibru Saikhowa (c) Sesachalam Hills, Achanakmar-Amarkantak, Dihang Dibang (d) Nilgiri, , Dihang Dibang

Answer: (c) Explanation: 1. Seshachalam: The hill ranges spread in the parts of Chittoor and districts of Andhra Pradesh have been designated as Seshachalam Biosphere Reserve. 2. Achanakmar-Amarkantak: This biosphere reserve is located in Chhattisgarh and Madhya Pradesh State. It is home to tigers, panthers, bears, Chitals, sambars, barking deers and bison (). 3. Dihang Dibang: Biosphere reserve located in Arunachal Pradesh exhibits natural vegetation stretching in an unbroken sequence from the tropics to the mountain tundra. Value Addition: Dibru Saikhowa: This Biosphere Reserve is located in the flood plains of Brahmaputra plains of Assam. It is located in one of the 19 biodiversity hotspots of the world. Nilgiri: It was the first biosphere reserve in India established in the year 1986.The reserve is situated in the Western Ghats, in the Nilgiri Hills of South India and is considered as an International Biosphere Reserve. The Nilgiri Biosphere Reserve was declared as World Heritage site in 2012.

Prelim IAS Test Series (2019) – GS Test 12 (24.02.2019) 41 Geography 2 and Current Affairs Sep, Oct, Nov, Dec 2018

Pachmarhi: This Biosphere Reserve is located in Central India. The Satpura mountain ranges cross India from west to east and Pachmarhi lies directly at its centre. Source: Orient Black Swan School Atlas, Page no. 35

70. With respect to Census 2011 of India, consider the following statements: (1) Decadal sex ratio of total population has continuously decreased since 1961. (2) Decadal sex ratio of children aged 0-6 years has continuously increased since 1961. Which of the statements given above is/are not correct? (a) 1 only (b) 2 only (c) Both 1 and 2 (d) Neither 1 nor 2

Answer: (c) Explanation: Statement 1 is incorrect: Decadal Sex ratio of total population increased in 1981, 1991 and 2011. Statement 2 is incorrect: Decadal Sex ratio of children aged 0-6 years has continuously decreased since 1961.

71. Generally, deltas are seen on the eastern coast and not on the western coast of India. Which of the following factors are mainly responsible for such contrasting feature of the Indian coasts? (1) East flowing rivers flow over gentle slope of Indian peninsula. (2) East flowing rivers have a short course. (3) East flowing rivers carry huge loads of sediments. (4) East flowing rivers flows through a rift valley. Select the correct answer using the code given below: (a) 1 and 2 only (b) 1 and 3 only (c) 2 and 4 only (d) 1, 2 and 4 only

Answer: (b) Explanation: Factor 1 is correct. Slight tilting of the peninsular block from northwest to the south-eastern direction gave an orientation to the entire drainage system towards the Bay of Bengal. Gentle slope means more deposits thus bigger deltas. Such rivers flow with lesser velocity before entering into sea.

Prelim IAS Test Series (2019) – GS Test 12 (24.02.2019) 42 Geography 2 and Current Affairs Sep, Oct, Nov, Dec 2018

Factor 2 is incorrect. Short course of rivers doesn’t provide much scope to collect large amount of slit. Generally, east flowing rivers like Godavari, Krishna which form deltas have a long course. Factor 3 is correct. East flowing rivers have enough tributaries and big catchment area. Hence, they are laden with huge amount of sediments eroded by such tributaries over their long course. Factor 4 is incorrect. Very few east flowing rivers traverse through rift valley. Rift Valley swiftly drains whatever sediments are carried by such rivers into the sea without forming any delta. Source: Vajiram and Ravi - Yellow Book, Indian Geography, Chapter 3: Drainage system and River Water, Page no. 42

72. Identify the type of Indian forest indicated by “X” marks in the following map:

(a) Moist deciduous forest (b) Montane forest (c) Tropical evergreen forest (d) Littoral forest

Answer: (c) Explanation: Tropical Evergreen Forest: These forests are found in the hills of the north-eastern region, western slope of the Western Ghats and the Andaman and Nicobar Islands. They are found in warm and humid areas with an annual precipitation of over 200 cm and mean annual temperature above 22֯C. The semi-evergreen forests are also found in the lesser rainy parts of these regions. Such forests have a mixture of evergreen and moist deciduous trees. Source: NCERT India Physical Environment Class XI – Chapter 5, page no. 57-58

73. Which of the following soil types is mainly developed in the areas of Peninsular Plateau experiencing high rainfall and high temperature? (a) Red Soil (b) Peaty Soil (c) Alluvial Soil (d) Laterite Soil

Answer: (d) Explanation: The laterite soils develop in areas with high temperature and high rainfall. These are the result of intense leaching due to tropical rains. These soils have mainly developed in the higher areas of the peninsular plateau. The laterite soils are commonly found in Karnataka, Kerala, Tamil Nadu, Madhya Pradesh and the hilly areas of Odisha and Assam. Laterites are not suitable for cultivation; however, application of manures and fertilisers are required for making the soils fertile for cultivation. Source: NCERT XI – India: Physical Environment, Chapter 6: Soils, Page no 71

Prelim IAS Test Series (2019) – GS Test 12 (24.02.2019) 43 Geography 2 and Current Affairs Sep, Oct, Nov, Dec 2018

74. Which of the following can act as serious development challenges to the Indian economy? (1) Inequality between different states (2) Inequality within the states themselves (3) Inequality between the rural and urban areas (4) Inequality within the rural societies and the urban societies themselves Select the correct answer using the code given below: (a) 1 and 2 only (b) 1 and 3 only (c) 1, 2 and 3 only (d) 1, 2, 3 and 4

Answer: (d) Explanation: With its wide diversities in physiography, history, demography and sociology, India has been characterised by regional disparities in socioeconomic development not only between States but also between districts of a State and between areas and social groups within districts. Following are the basic types of regional disparities in India:  Inter-state inequality (Developed Maharashtra state vs Underdeveloped North-Eastern states)  Inequality within the states themselves (Underdeveloped region within developed Maharashtra state)  Inequality between the rural and the urban areas (Underdeveloped vs developed Mumbai)  Inequality within the rural societies and the urban societies themselves Such regional disparities present a serious development challenge to the Indian economy. They hinder the progress towards ‘Faster, More Inclusive and Sustainable Growth’. Regional disparities in India can act as a counter current to the ideas of ‘National Integration and Federalism’ thus adversely affect the socio-economic indicators of the whole country. Source: Vajiram and Ravi – Yellow Book, Indian Geography – Chapter 16: Regional Development and Planning, Page no. 357

75. Consider the following statements: (1) Generally, El Nino and the Indian Monsoon are inversely related. (2) A negative Indian Ocean Dipole leads to less rainfall and more monsoon breaks in India. Which of the statements given above is/are correct? (a) 1 only (b) 2 only (c) Both 1 and 2 (d) Neither 1 nor 2

Answer: (c) Explanation: Statement 1 is correct: Generally, El Nino and the Indian Monsoon are inversely related. Conventional El Niño is characterized by strong anomalous warming in the eastern

Prelim IAS Test Series (2019) – GS Test 12 (24.02.2019) 44 Geography 2 and Current Affairs Sep, Oct, Nov, Dec 2018 equatorial Pacific. During El-Nino event, warming of the Pacific Ocean results in the weakening of Trade winds. Therefore, moisture and heat content gets limited and results in reduction and uneven distribution of rainfall across the Indian sub-. Statement 2 is correct: The Indian summer monsoon rainfall is influenced by a system of oscillating sea surface temperatures known as the Indian Ocean Dipole (IOD) in which the western Indian Ocean becomes alternately warmer and then colder than the eastern part of the ocean. A positive IOD occurs when the sea surface temperatures are greater than normal in the Arabian Sea and less than normal in the tropical eastern Indian Ocean. A positive IOD leads to greater monsoon rainfall and more active (above normal rainfall) monsoon days while negative IOD leads to less rainfall and more monsoon break days (no rainfall). Source: (1) NCERT XI – India: Physical Environment, Chapter 4: Climate, Page no. 38 (2) https://www.thehindu.com/sci-tech/science/The-dipole-factor-in-summer-monsoon- rainfall/article14479847.ece

76. Recently, Munroe Thuruthu Island was in news. It is reported to be sinking due to rising sea levels attributed to climate change. This island is located in (a) Andaman and Nicobar Islands (b) Tamil Nadu (c) Kerala (d) Lakshadweep

Answer: (c)

Explanation: Munroe Thuruthu in district, Kerala is slowly going under water, and its fleeing residents join the swelling ranks of climate refugees. Munroe Thuruthu is a string of eight islets at the confluence of the and the . The island can be seen as the first casualty of global warming in Kerala, creating a band of environmental refugees. The island’s decline began with the construction of the dam. Located 70 km away and constructed in the 1960s under the Kallada Irrigation Project, the dam blocked the flow of fresh water as a result of sediments from the Kallada river — the main determinant of the land’s fertility. Today the whole area has turned saline.

Prelim IAS Test Series (2019) – GS Test 12 (24.02.2019) 45 Geography 2 and Current Affairs Sep, Oct, Nov, Dec 2018

77. Consider the following pairs of Volcanoes (which were in news recently) and their respective location: Volcano Location 1. Mt. Krakatoa : Indonesia 2. Mt. Etna : Cyprus 3. Mt. Soputan : Italy Which of the pairs given above is/are correctly matched? (a) 1 only (b) 1 and 2 only (c) 2 and 3 only (d) 1, 2 and 3

Answer: (a) Explanation: 2018 witnessed some of the deadliest volcanic eruptions. Following volcanoes were in news recently: ● Mt. Krakatoa: Indonesia ● Mt. Etna: Italy ● Mt. Soputan: Indonesia Hence, only pair 1 is correctly matched.

78. ‘Susu’, normally found in the freshwaters of the River Ganga, are of great significance because (a) They signal river floods by making sound. (b) They remove the sediments from the river. (c) They eat away the algal bloom of a river. (d) They act as bio-indicators of aquatic pollution.

Answer: (d) Explanation: Top predators, such as mammalian carnivores, sea birds, and raptors, are among the widely used indicator species. Top predators tend to be concentrated in important biodiversity hotspots. The reduction or disappearance of top predators is related to significant ecosystem transformations, including impacts on several trophic levels and changes in energy flows, over-exploitation of resources, and changes in the behaviour of prey or food chain structure. Moreover, their presence or absence can indicate the extent of the footprint of human pressures. River dolphins, also known as 'susu' are top predators and can act as bio-indicators of aquatic pollution of freshwater on the basis of three ecological indicators: (i) density of river dolphins, (ii) mean group size of dolphins, and (iii) dolphin sighting rates.

79. Which of the following is responsible for nearly 65% of CO2 emissions in India? (a) Natural Gas (b) Cement (c) Oil (d) Coal

Prelim IAS Test Series (2019) – GS Test 12 (24.02.2019) 46 Geography 2 and Current Affairs Sep, Oct, Nov, Dec 2018

Answer: (d) Explanation: Although, India is rapidly going in for solar and wind power, coal usage continues to grow strongly. Coal is responsible for 65% of India's CO2 emissions. Limiting global warming to the 2015 Paris Agreement goal of keeping the global temperature increase this century, to well below 2°C, would need carbon dioxide emissions to decline by 50% by 2030 and reach net zero by about 2050.

80. All oral polio vaccines (OPV) across the world contain only two of the three polio serotypes - Type 1 and Type 3 where as Type 2 is banned. Which of the following statements is/are the primary reasons behind such a ban? (1) The wild Type 2 virus was eradicated globally by 1999. (2) Oral Polio Virus itself can cause polio in rare cases. Select the correct answer using the code given below: (a) 1 only (b) 2 only (c) Both 1 and 2 (d) Neither 1 nor 2

Answer: (c) Explanation: Polio vaccines are vaccines used to prevent poliomyelitis (polio). Two types are used: an inactivated poliovirus given by injection (IPV) and a weakened poliovirus given by mouth (OPV). All oral polio vaccines (OPV) across the world contain only two of the three polio serotypes — Type 1 and Type 3 where as Type 2 is banned. Reason 1 is correct. Type 2 is banned because the wild, disease-causing version of this virus was eradicated globally by 1999. However, sometime in September 2018, routine surveillance detected the Type 2 vaccine virus in stool samples from children in Uttar Pradesh, implying that someone was still making the vaccine. Further investigations revealed that the OPV, made by a Ghaziabad-based firm called Bio-Med, contained traces of the Type 2 vaccine virus. Reason 2 is correct. OPV itself can cause polio in rare cases. There are two ways in which all three oral vaccine viruses can cause polio. The first is called Vaccine Associated Paralytic Polio (VAPP). The second way in which the vaccine can cause polio is through Circulating Vaccine Derived Polio Virus (cVDPV).

81. The ‘Food Sustainability Index’ is based on the following three broad parameters: (1) Food loss and waste (2) Sustainable agriculture (3) Nutritional challenges As per comparison with the global scenario, under which parameter(s) mentioned above, India has performed well? (a) 1 only (b) 2 only (c) 2 and 3 only (d) 1, 2 and 3

Prelim IAS Test Series (2019) – GS Test 12 (24.02.2019) 47 Geography 2 and Current Affairs Sep, Oct, Nov, Dec 2018

Answer: (a) Explanation: A 'food system' involves the infrastructure and processes that go into feeding the population, such as growing, harvesting and transportation. The Food Sustainability Index (FSI) measures how sustainable these processes are. Despite the increase in food production, the FSI ranks India 33rd among 67 countries in 2018. China does better than India, while other BRICS partners are rated poorly. A comparison with the global scenario indicates following things:  India does well on preventing food loss and wastage. Hence, Point 1 is correct  It is below average in sustainable agriculture. Hence, Point 2 is incorrect  India is one of the worst countries in terms of tackling nutritional challenges. Hence, Point 3 is incorrect

82. Consider the following statements: (1) Among OPEC nations, though Venezuela has the most ‘Proven Reserves of oil’; produces the most crude oil per day. (2) In the year 2018, 's ‘Proven Reserves of natural gas’ have become more than that of Russia. Select the correct answer using the code given below: (a) 1 only (b) 2 only (c) Both 1 and 2 (d) Neither 1 nor 2

Answer: (a) Explanation: Statement 1 is correct. Saudi Arabia has the second-largest reserves of naturally occurring oil in the world after Venezuela. The Kingdom of Saudi Arabia produces a whopping 13.24 percent of all the oil produced daily in the entire world. Saudi Arabia and Russia top the list of oil exporting countries but Russia is not a member of (Organization of the Petroleum Exporting Countries) OPEC. Statement 2 is incorrect. Nearly 80% of the world's total proven natural gas reserves are located in ten countries. Russia tops the list, holding about a quarter of world's total gas reserves, followed by Iran and Qatar in the .

83. Recently, the term ‘Sharp Eye’ was in news which highlighted India’s progress in the field of indigenous technology. It denotes (a) An infra-red imaging drone developed by DRDO for surveillance of sensitive area near Line of Control (b) The facial expression recognition algorithm developed by IIT Chennai for artificial intelligence related application. (c) The most powerful camera ever developed by the ISRO to be used in space satellite. (d) None of the above

Answer: (c) Explanation:

Prelim IAS Test Series (2019) – GS Test 12 (24.02.2019) 48 Geography 2 and Current Affairs Sep, Oct, Nov, Dec 2018

India's Hyper Spectral Imaging Satellite (HysIS) is dubbed as ‘Sharp Eye’. HysIS is the best-ever high-resolution earth imaging satellite launched into space by India. HysIS is a very rare satellite with a super-sharp eye, and very few countries have indigenously mastered this technology. "Sharp eye" denotes the most powerful camera ever developed by the ISRO for use in space. HysIS will enable better identification of objects on the surface of the subcontinent, and aid in forestry, agriculture and smart city mapping. Only the United States, China and the European Space Agency have systems as sophisticated as our hyper-spectral imaging camera. HysIS satellite will improve both day and night viewing capabilities, giving India an edge over other countries.

84. Adequacy of Foreign Exchange Reserve of a country is measured by (a) Debt to GDP Ratio (b) Import Cover (c) Export Cover (d) Debt to Foreign Exchange Reserve Ratio

Answer: (b) Explanation: Import cover of reserves is the traditional trade-based indicator of foreign exchange reserve adequacy. It tells us how long imports can be sustained in the event of a shock. According to the IMF, traditionally, the import coverage measure has been based on months of prospective imports, with three months’ coverage typically used as a benchmark. As per , the reserves, which stood at $424.54 billion as at end-March 2018, fell to $405.74 billion as at end-June 2018. Latest data showed the reserves were at $392.7 billion by end of November 16, 2018

85. According to the findings of 'Global Carbon Project', which among the following are the four biggest CO2 emitters? (a) China, U.S., India, Japan (b) China, U.S., India, Germany (c) China, U.S., European Union, India (d) China, U.S., India, Russia

Answer: (d) Explanation: Global carbon emissions are set to hit an all-time high of 37.1 billion tonnes of CO2 in 2018 - 2019, according to researchers at the University of East Anglia (UEA) and the Global Carbon Project. The 10 biggest emitters in 2018 are China, U.S., India, Russia, Japan, Germany, Iran, Saudi Arabia, South Korea, and Canada. The EU as a region of countries ranks third. China’s emissions accounted for 27% of the global total, having grown an estimated 4.7% in 2018 and reaching a new all-time high.

Prelim IAS Test Series (2019) – GS Test 12 (24.02.2019) 49 Geography 2 and Current Affairs Sep, Oct, Nov, Dec 2018

86. GSAT-11 is India’s first six-tonne-class advanced satellite. It was put into orbit from the European spaceport in Guiana in South America. Its primary mission is to (a) boost communication and improve disaster links among India’s six neighbouring countries. (b) provide hyper-spectral imaging services for a range of applications like agriculture, forestry, etc. (c) monitor weather and climate of the Earth, especially global warming effects. (d) enable high-speed satellite-based internet services to users in rural, remote areas.

Answer: (d) Explanation: India’s first six-tonne-class ‘big bird’ in space, advanced communication satellite GSAT- 11, was put into orbit from the European spaceport in Guiana in South America. The heaviest ever to be built by the Indian Space Research Organisation (ISRO), the 5,854-kg satellite was launched from the Guiana Space Centre at Kourou. Its mission is to enable high-speed satellite-based Internet services to users in rural, remote areas and to businesses down home over the next 15 years. It will meet most of the requirements of providing broadband connectivity to rural and inaccessible village panchayats under Bharat Net which is part of the Digital India initiative.

87. Recently, the Reserve Bank of India has relaxed the securitization norms by relaxing the minimum holding period requirement. This step was mainly targeted to provide liquidity support to (a) Foreign banks (b) Real estate (c) Aviation Sector (d) Non-Banking Financial Institutions

Answer: (d) Explanation: In a move to make more liquidity available to non-banking finance firms, the Reserve Bank of India has relaxed the securitization norms by relaxing the minimum holding period requirement. The move follows a demand from the government for a special window for Non-banking financial Companies (NBFCs), to provide them liquidity support. The NBFC sector is facing liquidity shortage after Infrastructure Leasing & Finance Services, a core investment company, started defaulting on loans which resulted in the government dismantling the existing board of IL&FS and installing a new one. The cost of funds has gone up for the non-banking finance firms putting pressure on profitability.

88. Planets, stars and galaxies are just too far away to be analysed in a laboratory. Important aspects of such distant bodies can most effectively be studied with the help of (a) Solar Flares (b) Vacuum (c) Sound (d) Light

Prelim IAS Test Series (2019) – GS Test 12 (24.02.2019) 50 Geography 2 and Current Affairs Sep, Oct, Nov, Dec 2018

Answer: (d) Explanation: Astronomical spectroscopy is the study of astronomy using the techniques of spectroscopy to measure the spectrum of electromagnetic radiation, including visible light, which radiates from stars and other celestial objects. A spectrograph is an instrument used to obtain and record an astronomical spectrum. The spectrograph disperses the light from an object into its component wavelengths so that it can be recorded then analysed. Recently, a team of Indian scientists and engineers discovered a sub-Saturn or super- Neptune size planet around a Sun-like star. The discovery was made using Spectrograph, PARAS to measure and confirm the mass of the new planet.

89. To encourage inter-tribal interaction and to promote cultural heritage, a 'Hornbill Festival' is organized every year in the first week of December. In which state of India, this unique festival is organized? (a) Nagaland (b) Assam (c) Tripura (d) Manipur

Answer: (a) Explanation: To encourage inter-tribal interaction and to promote cultural heritage of Nagaland, the Government of Nagaland organizes the Hornbill Festival every year in the first week of December. It is also called the ‘Festival of Festivals”. The aim of the festival is to revive and protect the rich culture of Nagaland and display its extravaganza and traditions. The Festival is named after the hornbill, the globally respected bird and which is displayed in folklore in most of the state’s tribes.

90. The Government of India has started implementation of Dam Rehabilitation and Improvement Project (DRIP) to rehabilitate 198 dam projects in 7 States with financial assistance from (a) Asian Infrastructure Investment Bank (b) Asian Development Bank (c) World Bank (d) New Development Bank

Answer: (c) Explanation: The Government of India has started implementation of Dam Rehabilitation and Improvement Project (DRIP) with financial assistance from World Bank to rehabilitate 198 dam projects in 7 States viz. Kerala, Madhya Pradesh, Odisha, Karnataka, Uttarakhand, and Tamil Nadu. Central Water Commission (CWC) maintains National Register of Large Dams (NRLD) as per information provided by State Govts/ Dam owners. Prime responsibility of preparation of Disaster Management Plan (DMP)/ Emergency Action Plan (EAP) rests with dam

Prelim IAS Test Series (2019) – GS Test 12 (24.02.2019) 51 Geography 2 and Current Affairs Sep, Oct, Nov, Dec 2018 owners which are State Governments, central and state PSUs, municipalities and private companies etc.

91. The term 'E-flow' was in news recently. It means (a) minimum business per year by an e-commerce company to avoid bankruptcy. (b) minimum flow requirements to maintain the river in a desired environmental state. (c) minimum investment requirement for an angel investor who wishes to invest in a new start up. (d) minimum time taken by receivers on the Earth to detect signals coming from the dark side of the Moon.

Answer: (b) Explanation: 'Environmental flows' or e-flows are the “minimum flow requirements” and as long as flow is kept above a critical level, the river ecosystem will be conserved. Environmental flows are the acceptable flow regimes that are required to maintain a river in the desired environmental state or predetermined state. The Central Government on October 10th 2018 notified the minimum environmental flows for River Ganga that has to be maintained at various locations on the river. The notification issued by the Government will ensure that the river has at least the minimum required environmental flow of water even after the river flow gets diverted by projects and structures for purposes like irrigation, hydropower, domestic and industrial use etc. This was an important step taken towards maintaining the uninterrupted flow of the river.

92. With reference to Eklavya Model Residential Schools (EMRs), consider the following statements: (1) For the promotion of education among scheduled caste (SC) students, EMRSs are set up in Indian States / Union Territories (UTs). (2) The establishment of EMRSs is based on demand of the concerned Indian States/UTs with availability of land as an essential attribute. Which of the statements given above is/are not correct? (a) 1 only (b) 2 only (c) Both 1 and 2 (d) Neither 1 nor 2

Answer: (a) Explanation: Statement 1 is incorrect: In the context of establishing quality residential schools for the promotion of education, Eklavya Model Residential Schools (EMRSs) for Scheduled Tribes (ST) students are set up in States / UTs with provisioning of funds through “Grants under Article 275(1) of the Indian Constitution”. Statement 2 is correct: The establishment of EMRSs are based on demand of the concerned States/UTs with availability of land as an essential attribute. As per the budget 2018-19, every block with more than 50% ST population and at least 20,000 tribal persons, will have an Eklavya Model Residential School by the year 2022.

Prelim IAS Test Series (2019) – GS Test 12 (24.02.2019) 52 Geography 2 and Current Affairs Sep, Oct, Nov, Dec 2018

93. 'Tropical Tasar (BDR10)' has a great significance for Indian Economy. It is a (a) Minor Forest Produce (MFP) for which Minimum Support Price (MSP) has been declared by Indian Government recently. (b) Race of a silkworm which is newly developed by the Central Silk Board of India. (c) Species of butterfly which is endemic to Western Ghats in India. (d) Variety of drought resistant rice which is newly developed by the National Rice Research Institute, India.

Answer: (b) Explanation: Central Silk Board (CSB) has notified newly developed races of silkworm seed of mulberry and Vanya silk for increasing the productivity of cocoons and to increase the income of the farmers engaged in sericulture. Silk worm breeds for specific agro-climatic condition are essential for increasing the productivity of cocoons. Tropical Tasar Silkworm (BDR10) race developed by CSB have 21% more productivity than the traditional Daba breed. Farmers can get upto 52 kg cocoons per 100 disease free layings (dfls). This silkworm breed will help the tribal farmers of Jharkhand, Chhattisgarh, Odisha, West Bengal, Andhra Pradesh, Maharashtra, Madhya Pradesh, Bihar, and Uttar Pradesh. Central Silk Board under the Ministry of Textiles is engaged in applied research for developing new breeds of races of silkworm seed and conducts extensive field trial before commercial use in the field.

94. Recently, an autonomous institute under Ministry of Science and Technology has developed oligosaccharides-based coating. This coating is primarily used to (a) Improve the stealth feature of fighter jets. (b) Improve the reflective potential of concrete building walls. (c) Enhance the absorption capacity of solar panels. (d) Enhance the shelf-life of fruits and vegetables.

Answer: (d) Explanation: National Agri-Food Biotechnology Institute (NABI) is under the administrative control of Department of Biotechnology, Ministry of Science and Technology, Government of India. NABI has developed oligosaccharides-based natural fruit coating that enhances the shelf- life of fruits and vegetables. The technology has dual advantages, first it utilizes the agriculture waste and second it can replace the shellac, a coating material that is obtained from an insect. Value Addition: NABI has demonstrated proof of concept in Banana for the feasibility of carrying out CRISPR based genome editing. This technology is being exploited to develop traits in Banana as well as in crops like wheat, rice and Lathyrus.

95. Consider the following statements:

Prelim IAS Test Series (2019) – GS Test 12 (24.02.2019) 53 Geography 2 and Current Affairs Sep, Oct, Nov, Dec 2018

(1) Inland Water Transport (IWT) mode is an environment friendly and cost effective mode of transport. (2) National Waterway-2 is a section of the Brahmaputra River located in the Indian state of Assam only. Which of the statements given above is/are correct? (a) 1 only (b) 2 only (c) Both 1 and 2 (d) Neither 1 nor 2

Answer: (c) Explanation: Statement 1 is correct: Inland Water Transport (IWT) mode is widely recognized as environment friendly and cost effective mode of transport. As per RITES Report of 2014 on Integrated National Waterways Transportation Grid, some of the important benefits of IWT mode compared to rail and road transportation are fuel efficiency and cost savings. Statement 2 is correct: National Waterway 2 (NW-2) is a section of the Brahmaputra River having a length of 891 km between the Bangladesh border near and Sadiya in Assam. It was declared as National Waterway No. 2 on 1 September 1988. The NW 2 is one of the major freight transportation waterway of north east India and the third longest Waterway.

96. The Union Cabinet has approved “The DNA Technology (Use and Application) Regulation Bill, 2018”. The Bill has been formulated for the regulation of the use and application of Deoxyribonucleic Acid (DNA) technology. Which of the following statements is correct about DNA? (a) DNA is a single stranded molecule. (b) DNA is not stable under alkaline conditions. (c) DNA is self-replicating. (d) DNA is not susceptible to UV damage.

Answer: (c) Explanation: Statement a is incorrect: DNA is a double-stranded molecule while RNA is a single stranded molecule. Statement b is incorrect: DNA is stable under alkaline conditions while RNA is not stable. Statement c is correct: DNA is self-replicating. RNA is synthesized from DNA on an as- needed basis. Statement d is incorrect: DNA is susceptible to UV damage. Compared with DNA, RNA is relatively resistant to UV damage. Value Addition: DNA and RNA perform different functions in humans. DNA is responsible for storing and transferring genetic information while RNA directly codes for amino acids and acts as a messenger between DNA and ribosomes to make proteins.

97. Consider the following statements:

Prelim IAS Test Series (2019) – GS Test 12 (24.02.2019) 54 Geography 2 and Current Affairs Sep, Oct, Nov, Dec 2018

(1) National Steel Policy 2017 has set ambitious target of 300 MT steel capacity by 2030-31. (2) Steel Authority of India Limited (SAIL) is the largest steel producer and the highest iron ore miner in India. Which of the statements given above is/are correct? (a) 1 only (b) 2 only (c) Both 1 and 2 (d) Neither 1 nor 2

Answer: (c) Explanation: Statement 1 is correct: National Steel Policy 2017 has set ambitious target of 300 MT steel capacity by 2030-31. Currently, the per capita steel consumption in India is 60kg. The policy seeks to raise it to 160Kg by 2030. SAIL with 1/6th of the proposed capacity is going to be a key contributor. Statement 2 is correct: Steel Authority of India Limited (SAIL) is a Maharatna company and the largest steel producer and also the highest iron ore miner in India. Steel produced by SAIL is used for railway tracks, defence grade steel for construction of Arjun battle tank and India’s space programme of Chandrayaan and Mangalyaan also use SAIL steel. SAIL supplied around 35400 metric tonnes of steel for the construction of recently inaugurated India’s longest bridge - Bogibeel Road-cum-Rail Bridge on the river Brahmaputra. This quantity is little more than 50% of the total quantity of steel supplied for the bridge.

98. ‘NAQUIM’ programme was in news recently. The main objective of this Indian programme is to (a) Enhance the capacity of States in ground water management and development. (b) Enhance the quality management framework at Bureau of Indian Standards. (c) Monitor the quality of air in Indian urban metro cities. (d) Monitor the improvement in the quality of education at elementary level.

Answer: (a) Explanation: The National Project on Aquifer Management (NAQUIM) is an initiative of the Ministry of Water Resources, River Development & Ganga Rejuvenation, Government of India, for mapping and managing the entire aquifer systems in the country. The vision of the National Project on Aquifer Management is to identify and map aquifers at the micro level, to quantify the available groundwater resources, and to propose plans appropriate to the scale of demand and aquifer characteristics, and institutional arrangements for participatory management. The NAQUIM programme for mapping of water bearing aquifers has been planned with an aim to enhance the capacity of states in Ground Water Management and Development.

99. Studying the health and composition of ‘Benthic Macro-Invertebrates’ communities is of a great significance because they (a) help in cleaning the oil spill contamination in ocean waters.

Prelim IAS Test Series (2019) – GS Test 12 (24.02.2019) 55 Geography 2 and Current Affairs Sep, Oct, Nov, Dec 2018

(b) are responsible for algal bloom in many polluted water bodies. (c) are often used as indicators of aquatic ecosystem health. (d) help in mapping the ocean floor topography with a great accuracy.

Answer: (c) Explanation: Benthic (meaning “bottom-dwelling”) macro-invertebrates are small aquatic animals and the aquatic larval stages of insects. They include dragonfly and stonefly larvae, snails, worms, and beetles. They lack a backbone, are visible without the aid of a microscope. Benthic macro-invertebrates are often found attached to rocks, vegetation, logs and sticks or burrowed into the bottom and sediments. Benthic macro-invertebrates are commonly used as indicators of the biological condition of water bodies. They are reliable indicators because they spend all or most of their lives in water, are easy to collect and differ in their tolerance to pollution. Macro-invertebrates respond to human disturbance in fairly predictable ways, often live for more than a year and, unlike fish, have limited mobility. As they cannot escape pollution, macro- invertebrates have the capacity to integrate the effects of the stressors to which they are exposed. Community characteristics - such as abundance, richness, diversity, evenness, and community composition - can be monitored to determine whether the community is changing over time due to natural or human-caused impacts.

100. Bogibeel bridge, India’s longest road-rail bridge has immense economic and strategic significance for the nation. This bridge is built over which river? (a) Narmada (b) Jhelum (c) Brahmaputra (d) Godavari

Answer: (c) Explanation: The Bogibeel bridge, India’s longest road-rail bridge is in the state of Assam. The bridge, which spans the River Brahmaputra between and Dhemaji districts of Assam, is of immense economic and strategic significance for the nation. This bridge reduces distances between Assam and Arunachal Pradesh. It would greatly enhance "ease of living" in this region. The construction of the bridge will be a major boost to improve logistical issues for the armed forces along the border in Arunachal Pradesh as India shares a nearly 4,000 km border with China.

Prelim IAS Test Series (2019) – GS Test 12 (24.02.2019) 56 Geography 2 and Current Affairs Sep, Oct, Nov, Dec 2018